Download as pdf or txt
Download as pdf or txt
You are on page 1of 70

American Academy of Orthopaedic Surgeons

2002 shoulder Self-Assessment Examination

1. Which of the following statements best describes why the ulnar nerve is most prone
to neuropathy at the elbow?

1- It has the least longitudinal excursion required to accommodate elbow range of


motion.
2- It is subjected to both compression and traction during elbow motion. Page | 1
3- It passes between two muscle heads as it enters the forearm.
4- The dimensions of the entrance of the cubital tunnel do not change with elbow
motion.
5- The vascular supply leaves a watershed area of diminished arterial supply.

PREFERRED RESPONSE: 2

DISCUSSION: The ulnar nerve is more prone to neuropathy than the radial or median nerves for
many reasons. It has the greatest longitudinal excursion required to accommodate elbow range
of motion, subjecting it to potential traction forces. The dimensions of the entrance of the cubital
tunnel change with elbow motion, potentially causing compression in flexion. For these two
reasons, the ulnar nerve is subjected to both compression and traction during elbow motion.
Although it passes between two muscle heads as it enters the forearm, so do the median and
radial nerves. Finally, the vascular supply is adequate because of the anastamoses between the
superior ulnar collateral artery, the posterior ulnar recurrent artery, and the inferior ulnar
collateral artery.

 Norris TR (ed): Orthopaedic Knowledge Update: Shoulder and Elbow. Rosemont, IL, American Academy of Orthopaedic
Surgeons, 1997, pp 369-378.
 Prevel CD, Matloub HS, Ye Z, Sanger JR, Yousif NJ: The extrinsic blood supply of the ulnar nerve at the elbow: An
anatomic study. J Hand Surg Am 1993;18:433-438.
 Gelberman RH, Yamaguchi K, Hollstein SB, et al: Changes in interstitial pressure and cross-sectional area of the cubital
tunnel and of the ulnar nerve with flexion of the elbow. J Bone Joint Surg Am 1998;80:492-501.

2. Figure 1 shows the radiograph of a


71-year-old man who has had
increasing pain and weakness in his
shoulder for the past 3 years.
Nonsurgical management has failed
to provide relief. Examination shows
130 degrees of active forward flexion
and intact external rotation strength.
During surgery, a 1- x 1-cm rotator
cuff tear involving the supraspinatus
is encountered. Treatment should
include
1- humeral head replacement with rotator cuff repair.
2- humeral head replacement without rotator cuff repair.
3- arthrodesis of the shoulder.
4- total shoulder replacement with rotator cuff repair.
5- total shoulder replacement without rotator cuff repair.

PREFERRED RESPONSE: 4

DISCUSSION: Given the size of the rotator cuff tear, it is likely to be repaired; therefore, the
treatment of choice is a total shoulder replacement with rotator cuff repair. Severe rotator cuff
insufficiency can lead to early glenoid failure because of superior instability, and glenoid
resurfacing should be avoided in those instances.

 Boyd AD Jr, Thomas WH, Scott RD, Sledge CB, Thornhill TS: Total shoulder arthroplasty versus hemiarthroplasty:
Indications for glenoid resurfacing. J Arthroplasty 1990;5:329-336.
 Arntz CT, Jackins S, Matsen FA III: Prosthetic replacement of the shoulder for treatment of defects in the rotator cuff and
surface of the glenohumeral joint. J Bone Joint Surg Am 1993;75:485-491.

3. Which of the following is considered the cause of Milwaukee shoulder, a joint


disease similar to rotator cuff arthropathy?

1- Abundance of basic calcium phosphate crystals


2- Abundance of calcium pyrophosphate crystals
3- Gout
4- Rheumatoid arthritis
5- Osteonecrosis

PREFERRED RESPONSE: 1

DISCUSSION: Neer and associates focused on mechanical and nutritional factors as the etiology
of rotator cuff arthropathy. McCarty and associates, in describing a similar syndrome known as
Milwaukee shoulder, focused on an inflammatory cause in proposing the pathogenic role of
hydroxyapatite, a basic calcium phosphate.

 Neer CS II, Craig EV, Fukuda H: Cuff-tear arthropathy. J Bone Joint Surg Am 1983;65:1232-1244.
 McCarty DJ, Halverson PB, Carrera GF, Brewer BJ, Kozin F: Milwaukee shoulder: Association of microspheroids
containing hydroxyapatite crystals, active collagenase, and neutral protease with rotator cuff defects. I: Clinical aspects.
Arthritis Rheum 1981;24:464-473.

4. The MRI scan of the shoulder shown in Figure 2 was performed with the arm in
abduction and external rotation. The image reveals what condition?

Dr. Ahmed Altaei


American Academy of Orthopaedic Surgeons
2002 shoulder Self-Assessment Examination

1- Contact between the rotator cuff


and the posterior-superior
labrum
2- Anterior instability
3- A ganglion cyst of the
spinoglenoid notch Page | 3
4- Osteonecrosis of the humeral
head
5- Posterior subluxation

PREFERRED RESPONSE: 1

DISCUSSION: Internal impingement of the shoulder is now a well-recognized cause of shoulder


pain in the throwing athlete. First described by Walch and associates, it involves contact of the
rotator cuff and labrum in the maximally externally rotated and abducted shoulder, such as in the
late cocking phase of the throwing motion. Schickendantz and associates have shown this
contact to be physiologic in most patients and becoming pathologic with repetitive overhead
activity.

 Schickendantz MS, Ho CP, Keppler L, Shaw BD: MR imaging of the thrower’s shoulder: Internal impingement, latissimus
dorsi/subscapularis strains, and related injuries. Magn Reson Imaging Clin N Am 1999;7:39-49.
 Walch G, Boileau P, Noel E, et al: Impingement of the deep surface of the supraspinatus tendon on the posterosuperior
glenoid rim: An arthroscopic study. J Shoulder Elbow Surg 1992;1:238-245.
 McFarland EG, Hsu CY, Neira C, O’Neil O: Internal impingement of the shoulder: A clinical and arthroscopic analysis. J
Shoulder Elbow Surg 1999;8:458-460.

5. Figure 3 shows the radiographs of a 32-year-old man who fell 12 feet onto his
outstretched arm and sustained a fracture-dislocation of the elbow. Initial
management consisted of closed reduction of the dislocation. Surgical treatment
should now include repair or reduction and fixation of the

1- medial and lateral


collateral ligaments, radial
head, and coronoid.
2- medial collateral ligament
and coronoid.
3- lateral collateral ligament
and radial head.
4- medial and lateral
collateral ligaments.
5- radial head and coronoid.

PREFERRED RESPONSE: 1
DISCUSSION: The radiographs show fractures of the coronoid and radial head. The medial
collateral ligament has been avulsed from the ulnar insertion, and there is a valgus opening on
the medial side. The lateral collateral ligament is always disrupted in elbow dislocations and
fracture-dislocations that occur secondary to falls. This is known as the terrible triad injury
(dislocation and fractures of the coronoid and radial head); it has a very poor prognosis because
of its propensity for recurrent or persistent instability and late arthritis. The principle in treating
this injury is to repair all of the injured parts or protect them with a hinged external fixator until
they heal.

 Norris TR (ed): Orthopaedic Knowledge Update: Shoulder and Elbow. Rosemont, IL, American Academy of Orthopaedic
Surgeons, 1997, pp 345-354.
 Kasser JR (ed): Orthopaedic Knowledge Update 5. Rosemont, IL, American Academy of Orthopaedic Surgeons, 1996, pp
283-294.
 O’Driscoll SW, Jupiter JB, King GJ, Hotchkiss RN, Morrey BF: The unstable elbow. Instr Course Lect 2001;50:89-102.

6. It is important to avoid which of the following exercises in the immediate


postoperative period after humeral head replacement for an acute four-part
fracture?

1- Pendulum exercises
2- External rotation with a stick
3- Passive forward elevation
4- Active forward elevation
5- Active range of motion of the elbow, wrist, and hand

PREFERRED RESPONSE: 4

DISCUSSION: It is critical to withhold active range of motion of the shoulder within the first 6
weeks after arthroplasty for acute fracture to prevent tuberosity avulsion. When radiographic
and clinical findings show that the tuberosities are healed, active motion may be instituted,
usually at 6 to 8 weeks. Immediate passive range-of-motion exercises, including external
rotation with a stick, pendulum, and passive elevation, should begin within the limits of the
repair on the day of surgery to prevent stiffness.

 Hartstock LA, Estes WJ, Murray CA, et al: Shoulder hemiarthroplasty for proximal humerus fractures. Orthop Clin North
Am 1998;29:467-475.
 Hughes M, Neer CS: Glenohumeral joint replacment and postoperative rehabilitation. Phys Ther 1975;55:850-858.

7. A 38-year-old man has winging of the ipsilateral scapula after undergoing a


transaxillary resection of the first rib 3 weeks ago. What is the most likely cause of
this finding?

1- Persistent thoracic outlet syndrome


2- Injury to the upper trunk of the brachial plexus
3- Injury to the long thoracic nerve

Dr. Ahmed Altaei


American Academy of Orthopaedic Surgeons
2002 shoulder Self-Assessment Examination

4- Injury to the lower trunk of the brachial plexus


5- Injury to the spinal accessory nerve

PREFERRED RESPONSE: 3

DISCUSSION: During transaxillary resection of the first rib, the long thoracic nerve is at risk as Page | 5
it passes either through or posterior to the middle scalene muscle. Injury to this nerve may occur
as the result of overly aggressive retraction of the middle scalene during the procedure.

 Leffert RD: Thoracic outlet syndrome. J Am Acad Orthop Surg 1994;2:317-325.


 Todd TW: The descent of the shoulder after birth: Its significance in the production of pressure-symptoms on the lowest
brachial trunk. Anat Anz 1912;41:385-397.

8. A 73-year-old man who underwent repair of the left rotator cuff 6 years ago reports
good pain relief but notes residual weakness of the left shoulder, especially with
overhead tasks. He denies having pain at night and has minimal discomfort with
activities of daily living but is dissatisfied with his shoulder strength. Radiographs
show an acromiohumeral interval of 2 mm. Appropriate management should
consist of

1- an exercise program.
2- revision rotator cuff repair using local tissue transposition.
3- revision rotator cuff repair using allograft.
4- latissimus dorsi transfer.
5- combined latissimus dorsi and teres major transfer.

PREFERRED RESPONSE: 1

DISCUSSION: An exercise program to strengthen the deltoid and remaining rotator cuff will
most likely offer the best results. Revision rotator cuff surgery yields better results in decreasing
pain than improving strength and function, and this patient has only minimal pain. Tendon
transfers, involving the use of the latissimus dorsi or teres major, have been used when the
rotator cuff is deemed irreparable but are not indicated in elderly patients with minimal
symptoms.

 Bigliani LU, Cordasco FA, McIlveen SJ, Musso ES: Operative treatment of failed repairs of the rotator cuff. J Bone Joint
Surg Am 1992;74:1505-1515.
 DeOrio JK, Cofield RH: Results of a second attempt at surgical repair of a failed initial rotator-cuff repair. J Bone Joint
Surg Am 1984;66:563-567.
 Gerber C, Vinh TS, Hertel R, Hess CW: Latissimus dorsi transfer for the treatment of massive tears of the rotator cuff: A
preliminary report. Clin Orthop 1988;232:51-61.

9. A 45-year-old woman has had progressive right shoulder pain for the past 6 months.
She notes that the pain disrupts her sleep, she has pain at rest that requires the use
of narcotic analgesics, and she has limited use of her left shoulder for most activities
of daily living. History reveals the use of corticosteroids for systemic lupus
erythematosus. Examination shows diminished range of motion. Radiographs of
the right shoulder are shown in Figures 4a and 4b. Treatment should consist of

1- core decompression of the humeral head.


2- humeral arthroplasty.
3- total shoulder arthroplasty.
4- glenohumeral arthrodesis.
5- vascularized fibular allograft.

PREFERRED RESPONSE: 2

DISCUSSION: Humeral arthroplasty provides excellent pain relief and function for stage IV
osteonecrosis with humeral collapse. In late disease with glenoid involvement (stage V), total
shoulder arthroplasty is preferred. Some authors have reported satisfactory results with core
decompression of the humeral head for early stages of osteonecrosis, but results for stage IV
osteonecrosis are less satisfactory when compared with those for humeral arthroplasty.

 Cruess RL: Steroid-induced avascular necrosis of the head of the humerus: Natural history and management. J Bone Joint
Surg Br 1976;58:313-317.
 LePorte DM, Mont MA, Mohan V, Pierre-Jacques H, Jones LC, Hungerford DS: Osteonecrosis of the humeral head treated
by core decompression. Clin Orthop 1998;355:254-260.
 Neer CS II (ed): Shoulder Reconstruction. Philadelphia, PA, WB Saunders, 1990, pp 194-202.

10. The relocation test is most reliable for diagnosing anterior subluxation of the
glenohumeral joint when

1- posterior pressure placed on the humeral head results in increased pain.


2- external rotation with the arm in 90 degrees of abduction produces apprehension
that is relieved by posterior pressure on the humeral head.

Dr. Ahmed Altaei


American Academy of Orthopaedic Surgeons
2002 shoulder Self-Assessment Examination

3- external rotation with the arm in 90 degrees of abduction produces pain that is
relieved by posterior pressure on the humeral head.
4- external rotation with the arm in 90 degrees of abduction produces no symptoms,
but posterior pressure on the humeral head produces pain and apprehension.
5- external rotation with the arm in 90 degrees of abduction produces no symptoms,
but posterior pressure on the humeral head produces apprehension. Page | 7

PREFERRED RESPONSE: 2

DISCUSSION: The relocation test is most accurate when true apprehension is produced with the
arm in combined abduction and external rotation and then relieved when posterior pressure is
placed on the humeral head. Pain with this test is a less specific response and may occur with
other shoulder disorders such as impingement.

 Speer KP, Hannafin JA, Altchek DW, Warren RF: An evaluation of the shoulder relocation test. Am J Sports Med
1994;22:177-183.

11. A 16-year-old high school pitcher notes acute pain on the medial side of his elbow
during a pitch. Examination that day reveals medial elbow tenderness, pain with
valgus stress, mild swelling, and loss of extension. Plain radiographs show closed
physes and no fracture. Which of the following diagnostic studies will best reveal
his injury?

1- Technetium Tc 99m bone scan


2- Contrast-enhanced MRI
3- CT
4- Electromyography
5- Arthroscopy

PREFERRED RESPONSE: 2

DISCUSSION: The history and findings are consistent with a diagnosis of a sprain of the medial
collateral ligament (MCL) of the elbow; therefore, contrast-enhanced MRI is considered the
most sensitive and specific study for accurately showing this injury. Arthroscopic visualization
of the MCL is limited to the most anterior portion of the anterior bundle only; complete
inspection of the MCL using the arthroscope is not possible. CT without the addition of contrast
is of no value in this situation. Use of a technetium Tc 99m bone scan is limited to aiding in the
diagnosis of occult fracture, a highly unlikely injury in this patient. There are no clinical
indications for electromyography.

 Timmerman LA, Andrews JR: Undersurface tear of the ulnar collateral ligament in baseball players: A newly recognized
lesion. Am J Sports Med 1994;22:33-36.
 Timmerman LA, Schwartz ML, Andrews JR: Preoperative evaluation of the ulnar collateral ligament by magnetic resonance
imaging and computed tomography arthrography: Evaluation of 25 baseball players with surgical confirmation. Am J Sports
Med 1994;22:26-32.
 Fritz RC, Stoller DW: The elbow, in Stoller DW (ed): Magnetic Resonance Imaging in Orthopedics and Sports Medicine, ed
2. Philadelphia, PA, Lippincott Raven, 1995, pp 743-849.
12. Figures 5a and 5b show the radiographs of a 45-year-old patient. What is the most
likely diagnosis?

1- Glenoid dysplasia
2- Rheumatoid arthritis with centralization
3- Osteoarthritis with posterior glenoid wear
4- Posterior scapular fracture deformity
5- Traumatic posterior subluxation of the shoulder

PREFERRED RESPONSE: 1

DISCUSSION: Glenoid dysplasia is an uncommon anomaly that usually has a benign course but
may result in shoulder pain, arthritis, or multidirectional instability. Shoulder pain and instability
often improve with shoulder strengthening exercises.

 Wirth MA, Lyons FR, Rockwood CA Jr: Hypoplasia of the glenoid: A review of sixteen patients. J Bone Joint Surg Am
1993;75:1175-1184.
 Resnick D, Walter RD, Crudale AS: Bilateral dysplasia of the scapular neck. Am J Roentgenol 1982;139:387-390.

13. A 14-year-old boy sustains a twisting injury to his right shoulder and recalls feeling
a snap during a wrestling match. Examination shows hesitancy to raise the arm
away from the side, diffuse tenderness and swelling of the upper arm, and no
evidence of neurovascular compromise. Figures 6a and 6b show an AP radiograph
and MRI scan. What is the most likely diagnosis?

Dr. Ahmed Altaei


American Academy of Orthopaedic Surgeons
2002 shoulder Self-Assessment Examination

Page | 9

1- Anterior dislocation of the shoulder


2- Salter-Harris type I fracture of the proximal humeral physis
3- Rupture of the subscapularis tendon
4- Sprain of the acromioclavicular joint
5- Fracture of the glenoid neck

PREFERRED RESPONSE: 2

DISCUSSION: While difficult to appreciate on the AP radiograph of the shoulder, the increased
physeal signal demonstrated on the axial MRI scan is consistent with a nondisplaced growth
plate fracture. A comparison radiograph of the left shoulder also could be considered and the
injured shoulder evaluated for physeal widening.
Proximal humeral fractures in children are somewhat unusual, representing less than 1% of all
fractures seen in children and only 3% to 6% of all epiphyseal fractures. Physeal injuries are
classified according to the Salter-Harris classification scheme. Salter-Harris type I fractures
represent approximately 25% of physeal injuries to the proximal humerus in adolescents.
The proximal humeral physis is responsible for 80% of the longitudinal growth of the humerus;
therefore, there is tremendous potential for remodeling of fractures in this region. Management
for nondisplaced Salter-Harris type I fractures is limited to a short period of immobilization
followed by a gradual return to activities as clinical symptoms resolve.

 Curtis RJ, Rockwood CA Jr: Fractures and dislocations of the shoulder in children, in Rockwood CA Jr, Matsen FA III
(eds): The Shoulder. Philadelphia, PA, WB Saunders, 1990, pp 991-1007.
 Salter RB, Harris WR: Injuries involving the epiphyseal plate. J Bone Joint Surg Am 1963;45:587-622.

14. Figure 7 shows the radiograph of an otherwise healthy 65-year-old man who injured
his right dominant shoulder while skiing 18 months ago. He did not seek treatment
at the time of the injury. He now reports intermittent soreness when playing golf
but has no other limitations. Examination reveals full range of motion and no
tenderness, but he has slight pain with a crossed arm adduction stress test. He is
neurologically intact. Initial management should consist of

1- excision of the distal clavicle.


2- open reduction and internal fixation with intramedullary partial threaded pins.
3- open reduction and internal fixation with a reconstruction plate, screws, and bone
grafting.
4- bone grafting and use of heavy sutures to secure the clavicle to the coracoid.
5- observation and nonsteroidal anti-inflammatory drugs.

PREFERRED RESPONSE: 5

DISCUSSION: The radiograph shows a displaced type II distal clavicle fracture with nonunion.
Because the patient’s symptoms are minimal, the injury can be treated like a grade III
acromioclavicular separation. Present management should consist of ice, anti-inflammatory
drugs, activity modification, and perhaps physical therapy. If nonsurgical management fails to
provide relief, the surgical options are varied with no uniformity in the literature regarding
surgical treatment of this injury.

 Beaty JH (ed): Orthopaedic Knowledge Update 6. Rosemont, IL, American Academy of Orthopaedic Surgeons, 1999, pp
271-286.
 Craig EV: Fractures of the clavicle, in Rockwood CA Jr, Matsen FA III (eds): The Shoulder. Philadelphia, PA, WB
Saunders, 1998, vol 1, pp 428-482.

15. Figure 8 shows the AP radiograph of a 33-year-old woman


who sustained a midshaft clavicle fracture from a
motorcycle accident 15 months ago. She continues to have
significant pain with activities of daily living. Management
should consist of

1- use of an electrical bone stimulation unit.

Dr. Ahmed Altaei


American Academy of Orthopaedic Surgeons
2002 shoulder Self-Assessment Examination

2- open reduction and internal fixation with a dynamic compression plate placed
superiorly and autogenous bone grafting.
3- open reduction and internal fixation with a dynamic compression plate placed
inferiorly and autogenous bone grafting.
4- intramedullary screw fixation.
5- partial claviculectomy. Page | 11

PREFERRED RESPONSE: 2

DISCUSSION: The patient has a symptomatic painful atrophic midclavicular nonunion, and the
treatment of choice is rigid internal fixation with a dynamic compression plate and autogenous
bone grafting. A tension band effect is desired and achieved by placing the plate superiorly.
Excellent success rates of 90% to 100% have been reported using this technique. Intramedullary
screw fixation without bone grafting has a decreased success rate. Partial claviculectomy is not a
preferred option.

 Jupiter JB, Leffert RD: Non-union of the clavicle: Associated complications and surgical management. J Bone Joint Surg
Am 1987;69:753-760.
 Simpson NS, Jupiter JB: Clavicular nonunion and malunion: Evaluation and surgical management. J Am Acad Orthop Surg
1996;4:1-8.

16. A 62-year-old patient with rheumatoid arthritis has had pain and instability of the
elbow following total elbow replacement 2 years ago. A complete work-up,
including aspiration and cultures, is negative. Figures 9a and 9b show the AP and
lateral radiographs. Treatment should consist of

1- orthotic stabilization.
2- removal of the components with resection
arthroplasty.
3- revision total elbow arthroplasty with an
unconstrained prosthesis and ulnar allograft.
4- revision total elbow arthroplasty with a semiconstrained long-stemmed ulnar
prosthesis.
5- elbow arthrodesis with bone grafting.

PREFERRED RESPONSE: 4
DISCUSSION: The patient has aseptic loosening of the original semiconstrained prosthesis and
significant proximal ulnar bone destruction; therefore, the treatment of choice is revision
arthroplasty using a semiconstrained design. Although orthotic stabilization could be used, it
will not provide long-term pain relief. Resection arthroplasty after removal of the components
may lead to painful instability. Elbow arthrodesis would be difficult with the bone stock loss and
is not considered the best option. Two main contraindications to the use of an unconstrained
prosthesis are significant bone loss and previous use of a hinged or semiconstrained prosthesis.
An ulnar allograft could be combined with the use of a semiconstrained long-stemmed ulnar
prosthesis as a treatment modification.

 Ewald FC, Simmons ED Jr, Sullivan JA, et al: Capitellocondylar total elbow replacement in rheumatoid arthritis: Long-
term results. J Bone Joint Surg Am 1993;75:498-507.
 Morrey BF, King GJ: Revision of failed total elbow arthroplasty, in Morrey BF (ed): The Elbow and Its Disorders, ed 3.
Philadelphia, PA, WB Saunders, 2000, pp 602-610.

17. A 21-year-old football player reports increasing pain and a deformity involving his
chest after colliding with another player during a scrimmage. Imaging studies
confirm an anterior sternoclavicular dislocation. Management should consist of

1- reconstruction of the sternoclavicular capsule.


2- symptomatic nonsurgical treatment.
3- medial clavicle excision.
4- medial clavicle excision with capsular imbrication.
5- medial clavicle excision and rhomboid ligament reconstruction.

PREFERRED RESPONSE: 2

DISCUSSION: For the patient with an anterior sternoclavicular dislocation, the most appropriate
initial treatment should be symptomatic. Surgical options are usually contraindicated because
the incidence of intraoperative and postoperative complications is high. A deformity from an
anterior sternoclavicular dislocation is usually well tolerated. Return to play is allowed when
symptoms resolve.

 Rockwood CA Jr: Disorders of the sternoclavicular joint, in Rockwood CA Jr, Matsen FA III (eds): The Shoulder.
Philadelphia, PA, WB Saunders, 1998, vol 1, pp 477-525.
 Rockwood CA Jr, Odor JM: Spontaneous atraumatic anterior subluxation of the sternoclavicular joint. J Bone Joint Surg
Am 1989;71:1280-1288.

18. During total shoulder replacement for rheumatoid arthritis, fracture of the humeral
shaft occurs. An intraoperative radiograph shows a displaced short oblique
fracture at the tip of the prosthesis. At this point, the surgeon should

1- insert a standard humeral prosthesis with cerclage wires at the fracture site and
autologous cancellous bone graft.
2- insert a standard humeral component and apply a humeral orthosis
postoperatively.

Dr. Ahmed Altaei


American Academy of Orthopaedic Surgeons
2002 shoulder Self-Assessment Examination

3- cement a long-stemmed humeral component to bypass the fracture site and


supplement with cerclage wires.
4- remove all instrumentation, perform an open reduction and internal fixation of the
fracture, and delay completion of replacement surgery until the fracture has
healed.
5- discontinue the procedure and return for completion of total shoulder replacement Page | 13
when the fracture has healed.

PREFERRED RESPONSE: 3

DISCUSSION: The risk of intraoperative fracture in osteopenic rheumatoid bone is significant.


Fractures may occur with dislocation of the head and canal reaming, especially while extending
and externally rotating the shoulder. If the fracture occurs at the distal tip of the prosthesis, the
use of a long-stemmed prosthesis to bypass the fracture site and supplementation with wire
cables has been reported with good results.

 Wright TW, Cofield RH: Humeral fractures after shoulder arthroplasty. J Bone Joint Surg Am 1995;77:1340-1346.
 Boyd AD Jr, Thornhill TS, Barnes CL: Fractures adjacent to humeral protheses. J Bone Joint Surg Am 1992;74:1498-1504.
 Petersen SA, Hawkins RJ: Revision of failed total shoulder arthroplasty. Orthop Clin North Am 1998;29:519-533.

19. What is the most common contracture deformity of the spastic shoulder secondary
to a cerebrovascular accident?

1- External rotation, abduction, and extension


2- External rotation, adduction, and flexion
3- Internal rotation, abduction, and flexion
4- Internal rotation, adduction, and extension
5- Internal rotation, adduction, and flexion

PREFERRED RESPONSE: 5

DISCUSSION: The resultant spasticity and weakness (paresis) following a cerebrovascular


accident leads to muscle imbalance that commonly results in contracture of the shoulder in
adduction, internal rotation, and varying degrees of forward flexion. In addition, the elbow is
usually flexed and the forearm pronated.

 Braun RM, Botte MJ: Treatment of shoulder deformity in acquired spasticity. Clin Orthop 1999;368:54-65.
 McCollough NC III: Orthopaedic evaluation and treatment of the stroke patient. Instr Course Lect 1975;24:45-55.

20. A 21-year-old collegiate pitcher has had pain in his dominant shoulder for the past 3
months despite management consisting of rest, rehabilitation, and an analysis of
throwing mechanics. An arthroscopic photograph from the posterior portal is
shown in Figure 10. The biceps anchor to the bone was not detached to probing.
Treatment of the lesion to the left of the cannula should consist of arthroscopic
1- biceps tenodesis.
2- suture repair.
3- capsulorraphy.
4- debridement.
5- release of the biceps tendon.

PREFERRED RESPONSE: 4

DISCUSSION: The lesion is a variation of a type I superior labrum anterior and posterior lesion;
therefore, appropriate treatment is simple debridement. Biceps tenodesis or release is not
indicated because the biceps tendon and anchor are intact. There is no indication for labral repair
or capsulorraphy.

 Beaty JH (ed): Orthopaedic Knowledge Update 6. Rosemont, IL, American Academy of Orthopaedic Surgeons, 1999, pp
261-270.
 Snyder SJ, Banas MP, Karzel RP: An analysis of 140 injuries to the superior glenoid labrum. J Shoulder Elbow Surg
1995;4:243-248.

21. After humeral head replacement for four-part fractures, what is the most commonly
reported difficulty?

1- Pain
2- Inability to carry 10 lb at the side
3- Inability to wash the opposite axilla
4- Reaching to the back pocket
5- Working at shoulder level or above

PREFERRED RESPONSE: 5

DISCUSSION: Results show that patients who underwent humeral head replacement for fracture
almost routinely report pain relief, but functional reports vary. The most commonly reported
difficulty is the use of weight in the overhead position with wide variation in active elevation.
Factors found to affect active elevation include age, humeral offset, greater tuberosity
positioning, and four-part (as compared with three-part) fractures.

 Goldman RT, Koval KJ, Cuomo F, Gallagher MA, Zuckerman JD: Functional outcome after humeral head replacement for
acute three- and fourth-part proximal humeral fractures. J Shoulder Elbow Surg 1995;4:81-86.
 Hawkins RJ, Switlyk P: Acute prosthetic replacement for severe fractures of the proximal humerus. Clin Orthop
1993;289:156-160.

Dr. Ahmed Altaei


American Academy of Orthopaedic Surgeons
2002 shoulder Self-Assessment Examination

22. Figures 11a and 11b show the AP and lateral radiographs of a 32-year-old patient
on hemodialysis who has increasing elbow pain and a visibly growing mass over the
extensor surface. Figure 11c shows the photomicrograph of the biopsy specimen.
What is the most likely diagnosis?

Page | 15

1- Myositis ossificans
2- Tumoral calcinosis
3- Synovial cell sarcoma
4- Fungal granuloma
5- Hemochromatosis

PREFERRED RESPONSE: 2

DISCUSSION: The radiographic findings are classic for tumoral calcinosis; they are not
consistent with myositis ossificans, fungal granuloma, or hemochromatosis. The condition
typically appears as large aggregations of dense calcified lobules confined to the surrounding
soft tissues. Hyperphosphatemia is a fundamental factor in many patients with this condition.
Tumoral calcinosis also occurs in the setting of chronic renal failure when mineral homeostasis is
not controlled. The histologic appearance is essentially a foreign body granuloma reaction.
Multilocular cysts with purplish amorphous material are surrounded by thick connective tissue
capsules. The fibrous walls contain numerous foreign body giant cells. Surgical excision is
indicated if the tumor causes discomfort or interferes with function.

 Sisson HA, Murray RO, Kemp HBS (eds): Orthopaedic Diagnosis: Clinical, Radiological and Pathological Coordinates.
New York, NY, Springer-Verlag, 1984.
 Boskey AL, Vigorita VJ, Sencer O, Stuchin SA, Lane JM: Chemical, microscopic, and ultrastructural characterization of the
mineral deposits in tumoral calcinosis. Clin Orthop 1983;178:258-269.

23. A 52-year-old man who was a former high school pitcher now reports loss of elbow
flexion and extension with pain at the extremes of motion. Nonsurgical
management has failed to provide relief. Examination reveals movement from 50
degrees to 110 degrees and is painful only at the limits of motion. A radiograph is
shown in Figure 12. Treatment should consist of

1- excision of the osteophytes


and loose bodies and
anterior and posterior
capsular releases.
2- removal of the loose bodies.
3- anterior capsular release.
4- anterior and posterior
capsular releases.
5- interposition arthroplasty.

PREFERRED RESPONSE: 1

DISCUSSION: Based on the history, examination, and radiograph, the patient has typical
degenerative arthritis of the elbow. This condition is found almost exclusively in men, and there
is almost universally a history of repetitive heavy use or overuse of the elbow. Patients report
pain at terminal extension and usually have a flexion contracture. Radiographs reveal
osteophytes on the coronoid and olecranon and in the coronoid and olecranon fossae. The
osteophytes are often associated with loose bodies that sometimes are attached to the soft tissues.
Treatment should consist of removal of all loose bodies and impinging osteophytes using open
technique or by arthroscopy. The capsular contractures should be released at the same time.

 Kasser JR (ed): Orthopaedic Knowledge Update 5. Rosemont, IL, American Academy of Orthopaedic Surgeons, 1996, pp
283-294.
 Morrey BF: Primary degenerative arthritis of the elbow: Treatment by ulnohumeral arthroplasty. J Bone Joint Surg Br
1992;74:409-413.
 Redden JF, Stanley D: Arthroscopic fenestration of the olecranon fossa in the treatment of osteoarthritis of the elbow.
Arthroscopy 1993;9:14-16.
 O’Driscoll SW: Elbow arthritis: Treatment options. J Am Acad Orthop Surg 1993;1:106-116.

24. A 79-year-old woman with polyarticular rheumatoid arthritis has had progressively
increasing right shoulder pain for the past year, and nonsurgical management has
failed to provide relief. Her neurologic examination is entirely normal, but she is
unable to elevate her arm against gravity. An AP radiograph is shown in Figure 13.
Treatment should consist of

Dr. Ahmed Altaei


American Academy of Orthopaedic Surgeons
2002 shoulder Self-Assessment Examination

1- glenohumeral arthrodesis.
2- total shoulder arthroplasty.
3- humeral arthroplasty.
4- open synovectomy and rotator
cuff repair. Page | 17
5- anterior acromioplasty and
rotator cuff repair.

PREFERRED RESPONSE: 3

DISCUSSION: Because the patient has end-stage rheumatoid arthritis with glenoid and rotator
cuff deficiency, humeral arthroplasty is the treatment of choice. When a patient has an intact
rotator cuff and there is sufficient glenoid bone stock to implant a glenoid component, total
shoulder arthroplasty is the preferred method because it appears to provide more predictable pain
relief. Glenohumeral arthrodesis is generally avoided when there is a functional deltoid or
rotator cuff. Open synovectomy is appropriate in early rheumatoid disease before articular
changes are present. Anterior acromioplasty with coracoacromial ligament resection is avoided
in patients with rheumatoid arthritis because this procedure compromises the coracoacromial
arch and may result in anterosuperior instability.

 Neer CS II, Watson KC, Stanton FJ: Recent experience in total shoulder replacement. J Bone Joint Surg Am 1982;64:319-
337.
 Neer CS II: Glenohumeral arthroplasty, in Neer CS II (ed): Shoulder Reconstruction. Philadelphia, PA, WB Saunders,
1990, pp 143-271.
 Pollock RG, Deliz ED, McIlveen ST, et al: Prosthetic replacement in rotator cuff deficient shoulders. J Shoulder Elbow
Surg 1992;1:173-186.
 Sneppen O, Fruensgaard S, Johannsen HV, Olsen BS, Sojbjerg JO, Anderson NH: Total shoulder replacement in rheumatoid
arthritis: Proximal migration and loosening. J Shoulder Elbow Surg 1996;5:47-52.

25. A 22-year-old woman has had progressive upper


extremity weakness for the past several years.
History reveals no pain in her neck or shoulders.
Examination reveals scapular winging of both
shoulders and weakness in external rotation. She
can abduct to only 120 degrees bilaterally, and
there is mild supraspinatus weakness. She is
otherwise neurologically intact with normal
sensation and reflexes; however, she has difficulty
whistling. A clinical photograph is shown in
Figure 14. What is the most likely diagnosis?

1- Bilateral long thoracic nerve palsies


2- Central cervical disk herniation
3- Duchenne muscular dystrophy, adult onset
4- Fascioscapulohumeral dystrophy
5- Disuse atrophy as the result of deconditioning
PREFERRED RESPONSE: 4

DISCUSSION: Progressive weakness is a common sign with a large differential diagnosis.


Nerve, muscle, and joint problems should be excluded when a patient has diffuse weakness and
atrophy. Fascioscapulohumeral dystrophy is a rare disease characterized by facial muscle
weakness and proximal shoulder muscle weakness. The weakness is usually bilateral, and
scapular winging is common. If the scapular winging becomes pronounced, elevation of the
shoulder can be affected. In severe cases, scapulothoracic fusion or pectoralis muscle transfer to
the scapula may be indicated. Duchenne muscular dystrophy is typically severe and progressive.
The other diagnoses are not compatible with the history or the physical findings.

 Shapiro F, Specht L: The diagnosis and orthopaedic treatment of inherited muscular diseases of childhood. J Bone Joint
Surg Am 1993;75:439-454.
 Bunch WH, Siegel IM: Scapulothoracic arthrodesis in fascioscapulohumeral muscular dystrophy: Review of seventeen
procedures with three to twenty-one-year follow-up. J Bone Joint Surg Am 1993;75:372-376.

26. A 37-year-old electrician is diagnosed with a frozen shoulder after sustaining an


electrical injury at work 2 weeks ago. Examination reveals that he cannot actively or
passively externally rotate or abduct the arm. The glenohumeral joint and scapula
move in a 1:1 ratio. Radiographs are shown in Figures 15a and 15b. The best
course of action should be:

1- vigorous physical therapy for passive range of motion.


2- manipulation of the shoulder under anesthesia.
3- an intra-articular steroid injection.
4- an axillary radiograph.
5- MRI.

Dr. Ahmed Altaei


American Academy of Orthopaedic Surgeons
2002 shoulder Self-Assessment Examination

PREFERRED RESPONSE: 4

DISCUSSION: The patient’s history, examination, and radiographs are classic for locked
posterior dislocation of the glenohumeral joint. Posterior dislocation of the shoulder remains the
most commonly missed dislocation of a major joint. Up to 80% are missed on initial Page | 19
presentation. The primary cause for failure to accurately diagnose this injury is inadequate
radiographic evaluation. The typical presentation is a shoulder locked in internal rotation with
loss of abduction. An axillary view not only will make the definitive diagnosis but will help
assess the size of the articular surface defect and help plan treatment. This view can be done
expediently as part of every trauma series. The AP view is suspicious for a posteriorly
dislocated humerus with loss of the humeral neck profile, a vacant glenoid sign, and an anterior
humeral head compression fracture (reverse Hill-Sachs lesion).

 Hawkins RJ, Neer CS II, Pianta RM, Mendoza FX: Locked posterior dislocation of the shoulder. J Bone Joint Surg Am
1987;69:9-18.
 Norris TR (ed): Orthopaedic Knowledge Update: Shoulder and Elbow. Rosement, IL, American Academy of Orthopaedic
Surgeons, 1997, pp 181-189.
 Rockwood CA Jr, Thomas SC, Matsen FA III: Subluxations and dislocations about the glenohumeral joint, in Rockwood
CA Jr, Green DP, Bucholz RW (eds): Fractures in Adults, ed 3. Philadelphia, PA, JB Lippincott, 1991, vol 1, pp 1058-
1065.

27. An 80-year-old man has had increasing shoulder pain for the past 4 months. He
reports that it began with soreness and stiffness after chopping some wood. A
coronal MRI scan is shown in Figure 16. Initial management should consist of

1- shoulder exercises, mild


analgesics, and activity
modification.
2- transfer of the latissimus dorsi to
the greater tuberosity.
3- arthroscopy and debridement of
the tendon edges.
4- arthroscopy, arthroscopic
acromioplasty, coracoacromial
ligament release, and mini open
repair.
5- arthroscopy, arthrotomy,
acromioplasty, and primary repair
of the rotator cuff.

PREFERRED RESPONSE: 1

DISCUSSION: The MRI scan shows a massive tear of the supraspinatus tendon with medial
retraction to the level of the glenoid. This is most likely an attritional tear with a high risk of
failure of the repair. The preferred treatment is nonsurgical management for pain and stiffness.
Acromioplasty and coracoacromial ligament release in this setting are controversial, as they can
result in the devastating complication of anterosuperior subluxation of the humerus.

 Rockwood CA Jr, Williams GR Jr, Burkhead WZ Jr: Debridement of degenerative, irreparable lesions of the rotator cuff. J
Bone Joint Surg Am 1995;77:857-866.
 Beaty JH (ed): Orthopaedic Knowledge Update 6. Rosemont, IL, American Academy of Orthopaedic Surgeons, 1999, pp
299-312.

28. Figure 17 shows the radiograph of a 25-year-old professional football player who
has superior shoulder pain that prevents him from sports participation. History
reveals that he sustained a shoulder injury that was treated with closed reduction
and temporary pinning 3 years ago. The best course of action should be

1- no further participation in contact sports.


2- open reduction of the acromioclavicular joint and coracoclavicular screw
stabilization.
3- open repair of the coracoclavicular ligaments.
4- Weaver-Dunn reconstruction and coracoclavicular reconstruction.
5- excision of the distal clavicle.

PREFERRED RESPONSE: 4

DISCUSSION: The radiograph shows a complete acromioclavicular separation. Because the


patient is a professional athlete who is unable to participate, surgery is indicated. Chronic
separations, especially those with previous trauma from joint pinning, should be treated with
resection of the distal clavicle and stabilization to the coracoid. Some type of biologic
reconstruction of the coracoclavicular ligaments is generally recommended. Open repair of the
ligaments is generally not possible in such a delayed fashion. Screw fixation alone will not
provide a lasting solution as the screws usually need to be removed, leaving no fixation in place.
Reconstruction using the coracoacromial ligament is generally recommended with
coracoclavicular fixation to protect the repair while it heals.

Dr. Ahmed Altaei


American Academy of Orthopaedic Surgeons
2002 shoulder Self-Assessment Examination

 Nuber GW, Bowen MK: Disorders of the acromioclavicular joint: Pathophysiology, diagnosis and management, in Iannotti
JP, Williams GR (eds): Disorders of the Shoulder: Diagnosis and Management. Philadelphia, PA, Lippincott Williams and
Wilkins, 1999.
 Weinstein DM, McCann PD, McIlveen SJ, Flatow EL, Bigliani LU: Surgical treatment of complete acromioclavicular
dislocations. Am J Sports Med 1995;23:324-331.

Page | 21

29. A 54-year-old man undergoes total shoulder arthroplasty for osteoarthritis. Despite
compliance with an early passive range-of-motion exercise program, he does not
regain more than 90 degrees of elevation, 10 degrees of external rotation, and has
internal rotation to the fifth lumbar vertebra. At 6 months, his motion fails to
improve. Radiographs are shown in Figures 18a and 18b. What is the best course
of action?

1- Continue with a more aggressive passive


range-of-motion exercise program.
2- Perform an open release.
3- Revise the humeral component and
increase retroversion.
4- Revise the humeral component alone
after osteotomizing more of the humeral neck and seating the component lower.
5- Remove the glenoid component to decrease tension in the rotator cuff.

PREFERRED RESPONSE: 2

DISCUSSION: The patient has a global loss of motion that has failed to improve with 6 months
of nonsurgical treatment; because he has reached a plateau, further nonsurgical management will
likely be ineffective. Revision in the form of an open release is indicated to lyse intra- and extra-
articular adhesions; subscapularis lengthening may be done concurrently as needed. Revising to
a smaller head can be considered if adequate motion is not achieved. The radiographs reveal an
adequate neck cut with appropriate seating of the component. Removing the glenoid component
will decrease capsular tension but will probably increase pain because of the lack of glenoid
resurfacing. Increasing humeral retroversion will not improve motion.

 Cuomo F, Checroun A: Avoiding pitfalls and complication in total shoulder arthroplasty. Orthop Clin North Am
1998;29:507-518.
 Wirth MA, Rockwood CA Jr: Complications of shoulder arthroplasty. Clin Orthop 1994;307:47-69.
30. A 47-year-old patient has had persistent pain and weakness after undergoing a
reamed intramedullary nailing for a midshaft humerus fracture 8 months ago.
There is no evidence of infection. Radiographs are shown in Figures 19a and 19b.
Management should consist of

1- electrical stimulation.
2- retrograde nailing with multiple unreamed flexible nails to prevent further loss of
shoulder function.
3- leaving the same nail in place but adding cancellous bone graft.
4- exchange nailing with over-reaming and dynamic locking.
5- open reduction and plate fixation with autograft and rod removal.

PREFERRED RESPONSE: 5

DISCUSSION: Compression plating remains the treatment of choice for most established
humeral nonunions. Autograft is felt to be superior to allograft. Electrical stimulation has not
been found to improve healing rates in patients with nonunion after intramedullary nailing.
Retrograde nailing with flexible nails gives inadequate rotational control to promote healing in
this patient. Adding cancellous graft alone will not stabilize the nonunion site. Dynamic locking
has been successful only in the lower extremity because the bone can be loaded axially.

 McKee MD, Miranda MA, Riemer BL, et al: Management of humeral nonunion after the failure of locking intramedullary
nails. J Orthop Trauma 1996;10:492-499.
 Beaty JH (ed): Orthopaedic Knowledge Update 6. Rosemont, IL, American Academy of Orthopaedic Surgeons, 1999, pp
271-286.

31. An 18-year-old man sustained closed humeral shaft and forearm fractures of his
dominant arm in a motor vehicle accident. Neurovascular examination is intact,
and his condition is stable. The best course of action for management of the injuries
should be

Dr. Ahmed Altaei


American Academy of Orthopaedic Surgeons
2002 shoulder Self-Assessment Examination

1- external fixation of the forearm fracture and functional bracing of the humeral
shaft fracture.
2- external fixation of both fractures.
3- open reduction and internal fixation of both fractures.
4- open reduction and the internal fixation of the forearm fracture and functional
bracing of the humeral shaft fracture. Page | 23
5- application of a long arm cast.

PREFERRED RESPONSE: 3

DISCUSSION: Fractures above and below the elbow constitute floating elbow injuries and are
best treated with internal fixation to allow early range of motion and to prevent elbow stiffness.
Use of a long arm cast would promote elbow stiffness. External fixation is indicated primarily
for open injuries.

 Beaty JH (ed): Orthopaedic Knowledge Update 6. Rosemont, IL, American Academy of Orthopaedic Surgeons, 1999, pp
271-286.
 Bell MJ, Beachamp CG, Kellam JK, McMurtry RY: The results of plating humeral shaft fractures in patients with multiple
injuries: The Sunnybrook experience. J Bone Joint Surg Br 1985;67:293-296.

32. A 32-year-old woman has had pain and a visibly growing mass in the shoulder for 3
years but denies any history of trauma. Examination reveals a swollen, boggy
shoulder mass. The AP radiograph and MRI scan are shown in Figures 20a and
20b. Figures 20c through 20e show a portion of the excised mass and the
photomicrographs of the biopsy specimen. What is the most likely diagnosis?
1- Synovial chondromatosis
2- Pigmented villonodular synovitis
3- Synovial cell sarcoma
4- Tuberculosis
5- Chondrosarcoma

PREFERRED RESPONSE: 1

DISCUSSION: The radiographic findings are classic for synovial chondromatosis because of the
small calcified opacities within the joint surrounding the synovium. The histologic findings
show cartilaginous foci of metaplasia, which may be markedly cellular. However, unlike low-
grade chondrosarcoma, it lacks cellular and nuclear pleomorphism.

 Murphy FP, Dahlin DC, Sullivan CR: Articular synovial chondromatosis. J Bone Joint Surg Am 1982;44:77-86.
 Milgram JW: Synovial osteochondromatosis: A histopathological study of thirty cases. J Bone Joint Surg Am 1977;59:792-
801.

33. What is the most important factor regarding the risk of recurrent instability in a
patient with an acute anterior dislocation of the shoulder?

1- Age of the patient


2- Time from injury to reduction
3- Completion of 3 weeks of immobilization
4- The degree of athletic participation
5- Bilateral instability

PREFERRED RESPONSE: 1

DISCUSSION: The recurrence rate of anterior dislocation of the shoulder after the first episode
in athletes younger than age 20 years is thought to be as high as 90%, making surgery after the
initial episode a consideration. The rate drops from 50% to 75% in the 20- to 25-year age group
and down to 15% in patients older than age 40 years. An excellent prospective study of 257
patients in Sweden showed that there was no difference in those who did or did not complete 3
weeks of immobilization. The study also showed variability among different age groups in the
importance of athletic participation; athletes in the 12- to 22-year age group had a higher
recurrence rate, whereas the more sedentary patients in the 23- to 29-year age group had a higher
rate.

 Hovelius L: The natural history of primary anterior dislocation of the shoulder in the young. J Orthop Sci 1999;4:307-317.
 Simonet WT, Cofield RH: Prognosis in anterior shoulder dislocation. Am J Sports Med 1984;12:19-24.

34. A 25-year-old man injured his dominant shoulder after falling on his outstretched
arm 4 months ago. Examination reveals that he cannot lift his arm above 90 degrees,
and he has pronounced medial scapular winging. Management should consist of
Dr. Ahmed Altaei
American Academy of Orthopaedic Surgeons
2002 shoulder Self-Assessment Examination

1- spinal accessory nerve exploration with repair.


2- long thoracic nerve exploration with repair.
3- a sling for comfort, followed by shoulder strengthening exercises.
4- scapulothoracic arthrodesis.
5- split pectoralis major transfer.
Page | 25
PREFERRED RESPONSE: 3

DISCUSSION: Serratus anterior palsy or long thoracic nerve palsy is usually caused by traction
injury to the nerve, blunt injury, or iatrogenic injury. The palsy results in winging of the scapula
and medial rotation of the inferior pole of the scapula. A patient with this injury will usually
recover in 12 to 18 months. Initial treatment should include observation and shoulder
strengthening exercises. Nerve exploration with repair has not proven beneficial in changing the
outcome. Many orthopaedic surgeons favor using a split pectoralis major transfer for
symptomatic patients. Electrodiagnostic studies are helpful in confirming the diagnosis.

 Post M: Pectoralis major transfer for winging of the scapula. J Shoulder Elbow Surg 1995;4:1-9.
 Kuhn JE, Plancher KD, Hawkins RJ: Scapular winging. J Am Acad Orthop Surg 1995;3:319-325.

35. Treatment of adhesive capsulitis has a high failure rate when the underlying cause is

1- idiopathic.
2- traumatic.
3- diabetes mellitus.
4- hypothyroidism.
5- hyperthyroidism.

PREFERRED RESPONSE: 3

DISCUSSION: Diabetes mellitus has been associated with resistant cases of adhesive capsulitis.
With other causes of onset, adhesive capsulitis frequently responds to nonsurgical management
such as stretching exercises or, when this fails, manipulation under anesthesia and/or
arthroscopic release. Manipulation is rarely successful for the treatment of adhesive capsulitis
associated with diabetes mellitus, and arthroscopic release may be preferred.

 Fisher L, Kurtz A, Shipley M: Association between cheiroarthropathy and frozen shoulder in patients with insulin-
dependent diabetes mellitus. Br J Rheumatol 1986;25:141-146.
 Janda DH, Hawkins RJ: Shoulder manipulation in patients with adhesive capsulitis and diabetes mellitus: A clinical note. J
Shoulder Elbow Surg 1993;2:36-38.
 Pollock RG, Duralde XA, Flatow EL, Bigliani LU: The use of arthroscopy in the treatment of resistant frozen shoulder.
Clin Orthop 1994;304:30-36.
36. Figure 21 shows the AP radiograph of a 41-year-old patient who sustained a closed
bicolumnar fracture of the distal humerus that resulted in a painful nonunion.
What is the best initial construct for rigid stabilization of this fracture pattern?

1- Posterior “Y” plate fixation


2- Dual one third tubular plate fixation with a
hinged external fixator
3- Dual one third tubular plate fixation
4- Dual 3.5-mm reconstruction plate fixation
5- Single lateral plate fixation with transcortical
screw fixation

PREFERRED RESPONSE: 4

DISCUSSION: The dual plate fixation construct is significantly stronger than single plate or “Y”
plate fixation. Two-plate constructs at right angles, the ulnar plate medially and the lateral plate
posteriorly, would appear to be biomechanically optimal. This approach usually is feasible at the
time of surgery. Clinically, dual 3.5-mm reconstruction or dynamic compression plates are
superior to one third tubular plate fixation. Supplementary external fixation is not considered a
better treatment option. Failure of fixation and nonunion are often the result of inadequate
fixation and osteoporosis.

 Helfet DL, Hotchkiss RN: Internal fixation of the distal humerus: A biomechanical comparison of methods. J Orthop
Trauma 1990;4:260-264.
 Sodergard J, Sandelin J, Bostman O: Mechanical failures of internal fixation in T and Y fractures of the distal humerus. J
Trauma 1992;33:687-690.

37. Figure 22 shows the


radiographs of a 16-year-
old boy who injured his
elbow in a fall 1 year ago.
Although he has no pain,
he reports restricted
forearm rotation and elbow
flexion. What is the most
likely diagnosis?

Dr. Ahmed Altaei


American Academy of Orthopaedic Surgeons
2002 shoulder Self-Assessment Examination

1- Posttraumatic soft-tissue contractures


2- Congenital dislocation of the radial head
3- Chronic posttraumatic dislocation of the radial head
4- Combined annular and lateral collateral ligament injury
5- An unrecognized Monteggia variant type of injury
Page | 27
PREFERRED RESPONSE: 2

DISCUSSION: Congenital dislocation of the radial head is often confused with posttraumatic
dislocation. The distinguishing feature here is the dome-shaped radial head. Some patients with
congenital anomalies fail to recognize their limitations until an injury occurs. Soft-tissue
contractures do not cause radial head dislocation nor do they usually cause this pattern of motion
restriction (mainly flexion and rotation without significant loss of extension). There is no
deformity of the ulna to suggest an old Monteggia lesion.

 Morrey BF (ed): The Elbow and Its Disorders, ed 2. Philadelphia, PA, WB Saunders, 1993, p 196.
 Bell SN, Morrey BF, Bianco AJ Jr: Chronic posterior subluxation and dislocation of the radial head. J Bone Joint Surg Am
1991;73:392-396.

38. A 55-year-old man has had progressive right shoulder pain for the past 2 years.
Examination reveals active elevation to 120 degrees, external rotation to 20 degrees,
and internal rotation to the sacrum. AP and axillary radiographs are shown in
Figures 23a and 23b. Which of the following procedures would result in the most
predictable long-term pain relief?

1- Arthroscopic debridement of the glenohumeral joint


2- Open subscapularis lengthening and cheilectomy
3- Humeral hemiarthroplasty
4- Bipolar humeral hemiarthroplasty
5- Total shoulder arthroplasty
PREFERRED RESPONSE: 5

DISCUSSION: Total shoulder arthroplasty yields excellent pain relief and function in patients
with osteoarthritis. It is favored over humeral arthroplasty, especially when there is asymmetric
posterior glenoid wear and posterior humeral subluxation as shown on the axillary radiograph.
Arthroscopic debridement of the glenohumeral joint may be helpful in delaying the need for
arthroplasty when the arthritic changes are mild to moderate but is not indicated for advanced
osteoarthritis.

 Cofield RH, Frankle MA, Zuckerman JD: Humeral head replacement for glenohumeral arthritis. Semin Arthroplasty
1995;6:214-221.
 Levine WN, Djurasovic M, Glasson JM, Pollock RG, Flatow EL, Bigliani LU: Hemiarthroplasty for glenohumeral
osteoarthritis: Results correlated to degree of glenoid wear. J Shoulder Elbow Surg 1997;6:449-454.
 Matsen FA III: Early effectiveness of shoulder arthroplasty for patients who have primary glenohumeral degenerative joint
disease. J Bone Joint Surg Am 1996;78:260-264.

39. A 20-year-old professional baseball pitcher has had a 3-year history of increased
aching in his shoulder that is associated with pitching, and he is now seeking a
second opinion. Nonsurgical management consisting of rest, anti-inflammatory
drugs, ice, heat, and cortisone injections has failed to provide relief. A previous
work-up that included radiographs and gadolinium-enhanced MRI arthrography
was negative. Results of an arteriogram suggest quadrilateral space syndrome.
Assuming that this is the correct diagnosis, what nerve needs to be decompressed?

1- Suprascapular
2- Infraspinatus branch of the suprascapular
3- Long thoracic
4- Axillary
5- Lateral cord of the brachial plexus

PREFERRED RESPONSE: 4

DISCUSSION: Quadrilateral space syndrome is a rare condition and is the result of compression
of the contents of the quadrilateral space. The contents of the quadrilateral space include the
posterior circumflex vessels and the axillary nerve.

 Cahill BR, Palmer RE: Quadrilateral space syndrome. J Hand Surg 1983;8:65-69.
 Lester B, Jeong GK, Weiland AJ, Wickiewicz TL: Quadrilateral space syndrome: Diagnosis, pathology, and treatment. Am
J Orthop 1999;28:718-722.

40. A right-handed 24-year-old woman underwent an arthroscopic Bankart repair for


recurrent anterior dislocations 9 months ago. Despite extensive physical therapy for
8 months, the patient has very limited range of motion (elevation to 130 degrees and
external rotation to 10 degrees with the arm at the side). Shoulder radiographs are
normal. The next step in management should consist of

Dr. Ahmed Altaei


American Academy of Orthopaedic Surgeons
2002 shoulder Self-Assessment Examination

1- cessation of physical therapy and acceptance of the limited range of motion.


2- additional physical therapy for 3 to 4 months.
3- arthroscopic capsular release.
4- open release with Z-plasty lengthening of the subscapularis tendon.
5- closed manipulation under anesthesia.
Page | 29
PREFERRED RESPONSE: 3

DISCUSSION: Arthroscopic capsular release is an effective means of treating stiffness that is


the result of capsular contractures, such as in the case of a tight Bankart repair. Open release
allows lengthening of a surgically shortened subscapularis, such as after a tight Putti-Platt repair.
Additional physical therapy is unlikely to be effective because 8 months of treatment has failed
to result in improvement. Accepting this degree of asymptomatic limited motion is not advisable
because of the functional limitations for the patient and the increased risk of postoperative
degenerative arthritis.

 Warner JJ, Allen AA, Marks PH, Wong P: Arthroscopic release of postoperative capsular contracture of the shoulder. J
Bone Joint Surg Am 1997;79:1151-1158.
 Harryman DT II, Matsen FA III, Sidles JA: Arthroscopic management of refractory shoulder stiffness. Arthroscopy
1997;13:133-147.

41. A patient with deficient anteroinferior bone stock undergoes a Latarjet procedure
that transfers a portion of the coracoid to the glenoid rim and secures it with two
screws. After surgery, the patient reports numbness on the anterolateral forearm.
To verify the diagnosis, what muscle should be tested for strength?

1- Axillary
2- Abductor pollicis brevis
3- Supinator
4- Triceps
5- Biceps

PREFERRED RESPONSE: 5

DISCUSSION: A Latarjet procedure is similar to a Bristow procedure, but with the Latarjet
procedure a larger portion of the coracoid is transferred to the scapular neck at the anteroinferior
glenoid. As in a Bristow procedure, if the fragment is pulled or twisted during the dissection or
during fixation, the musculocutaneous nerve can be injured. With loss of biceps function, elbow
flexion and forearm supination will be weaker.

 Ho E, Cofield RH, Balm MR, Hattrup SJ, Rowland CM: Neurologic complications of surgery for anterior shoulder
instability. J Shoulder Elbow sdeSurg 1999;8:266-270.
 Boardman ND 3rd, Cofield RH: Neurologic complications of shoulder surgery. Clin Orthop 1999;368:44-53.
 Allain J, Goutallier D, Glorion C: Long-term results of the Latarjet procedure for the treatment of anterior instability of the
shoulder. J Bone Joint Surg Am 1998;80:841-852.
42. A 34-year-old woman has had painful snapping and popping in the elbow since
falling while in-line skating 6 months ago. The popping also occurs when she pushes
off with her hands to rise from a seated position. Initial radiographs were normal,
and she was told that she had sprained her elbow. Examination reveals few findings
except that she is very apprehensive when the forearm is forcefully supinated with
the elbow extended or partially flexed. A radiograph taken in that position is shown
in Figure 24. Treatment should consist of

1- arthroscopic loose body removal.


2- arthroscopic debridement and loose body removal for osteochondritis dissecans of
the capitellum.
3- annular ligament reconstruction for posttraumatic posterior subluxation of the
radial head.
4- radial head resection for congenital type II dislocation of the radial head.
5- lateral collateral ligament reconstruction for posterolateral rotatory instability.

PREFERRED RESPONSE: 5

DISCUSSION: The radiograph reveals posterolateral rotatory subluxation of the radiohumeral


and ulnohumeral joints. The space between the ulna and trochlea is enlarged, particularly
posteriorly at the olecranon. These findings are diagnostic of posterolateral rotatory instability,
which causes recurrent subluxation and reduction as the elbow is flexed from an extended and
supinated position with valgus load. The posterolateral rotatory instability apprehension test was
performed on this patient and the result was positive. The lateral pivot-shift test causes a clunk
as the elbow reduces but is more difficult to perform, even under general anesthesia. The patient
does not have isolated subluxation of the radial head, although these findings can be mistakenly
diagnosed as such. The radial head is normally shaped and does not represent a congenital
dislocation. There are no findings here to suggest osteochondritis dissecans or loose bodies.

 O’Driscoll SW, Bell DF, Morrey BF: Posterolateral rotatory instability of the elbow. J Bone Joint Surg Am 1991;73:440-
446.
 Burgess RC, Sprague HH: Post-traumatic posterior radial head subluxation: Two case reports. Clin Orthop 1984;186:192-
194.
 Norris TR (ed): Orthopaedic Knowledge Update: Shoulder and Elbow. Rosemont, IL, American Academy of Orthopaedic
Surgeons, 1997, pp 345-354.

Dr. Ahmed Altaei


American Academy of Orthopaedic Surgeons
2002 shoulder Self-Assessment Examination

43. A 49-year-old woman noted pain in her right axilla 1 day after moving heavy
furniture. Two weeks later, she now reports persistent numbness and paresthesias
along the inner aspect of her upper arm radiating into the ulnar digits.
Examination reveals full shoulder motion, tenderness over the first rib, and a
decreased radial pulse with the shoulder placed overhead. What is the most likely
diagnosis? Page | 31

1- Brachial plexus stretch injury


2- Cervical radiculopathy
3- Rotator cuff tendinitis
4- Anterior subluxation of the shoulder
5- Thoracic outlet syndrome

PREFERRED RESPONSE: 5

DISCUSSION: Thoracic outlet syndrome is thought to be caused by compression of the


neurovascular supply to the upper limb in the supraclavicular and axillary regions of the
shoulder. While typically progressive in onset, thoracic outlet syndrome may develop after acute
injury. Injury or weakness of the scapular muscles, especially the trapezius, may result in
descent of the scapula and cause compression of the thoracic outlet. In general, most symptoms
are the result of neural compression. Typical symptoms include pain in the neck or shoulder and
numbness or tingling that predominantly involves the ulnar side of the arm and hand.
Exacerbation of these symptoms is typical when the arm is abducted. Initial management should
consist of postural exercises aimed at restoring proper scapular stability. Severe recalcitrant
symptoms may warrant surgical decompression.

 Leffert RD: Thoracic outlet syndrome. J Am Acad Orthop Surg 1994;2:317-325.


 Todd TW: The descent of the shoulder after birth: Its significance in the production of pressure-symptoms on the lowest
brachial trunk. Anat Anz 1912;41:385-397.

44. A patient has had a locked posterior dislocation of the shoulder for the past 6
months. After undergoing total shoulder arthroplasty that includes adequate
anterior releases and posterior capsulorrhaphy, the patient still exhibits posterior
instability intraoperatively. The postoperative rehabilitation regimen should
include

1- use of a sling with no range-of-motion exercises until the condition is stable.


2- use of a sling and passive range-of-motion exercises within the limits of the repair.
3- no sling and supine passive range-of-motion exercises.
4- an internal rotation brace holding the arm at the side.
5- an external rotation brace holding the arm at the side.

PREFERRED RESPONSE: 5

DISCUSSION: Achieving stability in chronic locked posterior dislocations of the shoulder


remains a difficult challenge. Intraoperative measures include decreased humeral retroversion,
anterior releases, and posterior capsular tightening. Postoperative rehabilitation is of equal
importance. Immobilization in an external rotation brace (10 degrees to 15 degrees) with the arm
at the side for 4 to 6 weeks is recommended to decrease tension in the posterior capsule. When
passive range-of-motion exercises are instituted, they should be performed in the plane of the
scapula to avoid stress posteriorly. Internal rotation and supine elevation should be avoided for
similar reasons.

 Hawkins RJ, Neer CS II, Pianta RM, Mendoza FX: Locked posterior dislocation of the shoulder. J Bone Joint Surg Am
1987;69:9-18.
 Loebenberg MI, Cuomo F: The treatment of chronic anterior and posterior dislocations of the glenohumeral joint and
associated articular surface defects. Orthop Clin North Am 2000;31:23-24.

45. Which of the following factors is associated with failure of arthroscopic excision of
the distal clavicle?

1- Removal of less than 2 cm of bone


2- Male gender
3- Female gender
4- Diagnosis of osteolysis
5- Uneven resection of bone

PREFERRED RESPONSE: 5

DISCUSSION: Uneven resection of bone, typically leaving a retained posterolateral corner of


the distal clavicle, can lead to failure of arthroscopic distal clavicle excision. The amount of
bone resected, the gender of the patient, or the diagnosis (osteoarthritis versus osteolysis) does
not appear to affect the results.

 Flatow EL, Duralde XA, Nicholson GP, Pollock RG, Bigliani LU: Arthroscopic resection of the distal clavicle with a
superior approach. J Shoulder Elbow Surg 1995;4:41-50.

46. Anterior subluxation in a throwing athlete is most commonly the result of

1- avulsion of the inferior glenohumeral ligament from the glenoid.


2- avulsion of the inferior glenohumeral ligament from the humerus.
3- fracture of the anterior glenoid rim.
4- excessive capsular laxity from microtrauma.
5- a large Hill-Sachs lesion.

PREFERRED RESPONSE: 4

DISCUSSION: Subtle anterior subluxation in the throwing athlete most frequently results from
excessive capsular laxity because of repetitive microtrauma. Avulsion of the inferior
glenohumeral ligament from the glenoid, or more rarely from the humerus, occurs with
macrotrauma. A large Hill-Sachs lesion and a glenoid rim fracture also may result from a
traumatic anterior dislocation.

Dr. Ahmed Altaei


American Academy of Orthopaedic Surgeons
2002 shoulder Self-Assessment Examination

 Kvitne RS, Jobe FW: The diagnosis and treatment of anterior instability in the throwing athlete. Clin Orthop 1993;291:107-
123.
 Jobe FW, Tibone JE, Jobe CM, Kvitne RS: The shoulder in sports, in Rockwood CA Jr, Matsen FA III (eds): The Shoulder,
ed 2. Philadelphia, PA, WB Saunders, 1999, pp 961-990.

Page | 33

47. What is the most significant prognostic factor in nontraumatic osteonecrosis of the
humeral head?

1- Duration of symptoms
2- Age of the patient
3- Total amount of steroid use
4- Stage of the disease
5- Status of the rotator cuff

PREFERRED RESPONSE: 4

DISCUSSION: Use of systemic steroids has been implicated in the development of nontraumatic
osteonecrosis of the humeral head. Staging of the disease is most relevant to prognosis and
treatment. Cruess has described a widely accepted staging system. Several authors have shown
that patients who have a lower stage of disease (ie, stage I or II) have a much less likely chance
of progression compared with those who are in the later stages (IV and V).

 Cruess RL: Osteonecrosis of bone: Current concepts as to etiology and pathogenesis. Clin Orthop 1986;208:30-39.
 Cruess RL: Steroid-induced avascular necrosis of the humeral head: Natural history and management. J Bone Joint Surg Br
1976;58:313-317.
 Rutherford CS, Cofield RH: Osteonecrosis of the shoulder. Orthop Trans 1987;11:239.
 Hattrup SJ, Cofield RH: Osteonecrosis of the humeral head: Relationship of disease stage, extent, and cause to natural
history. J Shoulder Elbow Surg 1999;8:559-564.

48. A 43-year-old former professional hockey player reports severe pain in his chest
after being checked from the side in a pick-up hockey game. An MRI scan and
plain radiographs are shown in Figures 25a through 25c. What is the most likely
diagnosis?
1- Anterior sternoclavicular joint dislocation
2- Posteroinferior sternoclavicular joint dislocation
3- Anterior acromioclavicular joint dislocation
4- Posterior acromioclavicular joint dislocation
5- Acromial fracture

PREFERRED RESPONSE: 1

DISCUSSION: Anterior dislocation is the most common type of sternoclavicular dislocation.


The medial end of the clavicle is displaced anterior or anterosuperior to the anterior margin of
the sternum. In a study by Omer, 31% of athletic injuries have been known to cause a
dislocation of the sternoclavicular joint. The serendipity view can show this dislocation, as will
CT of the chest. This view requires the x-ray beam to be aimed at the manubrium with 40
degrees of cephalic tilt. An anterior sternoclavicular joint dislocation will appear superiorly
displaced, while a posterior sternoclavicular joint dislocation is inferiorly displaced on the
serendipity view.

 Rockwood CA Jr, Matsen FA III (eds): The Shoulder. Philadelphia, PA, WB Saunders, 1998, vol 1, pp 566-572.
 Omer GE Jr: Osteotomy of the clavicle in surgical reduction of anterior sternoclavicular dislocation. J Trauma 1967;7:584-
590.

49. Which of the following is considered a contraindication to functional bracing for the
treatment of humeral shaft fractures?

1- A closed midshaft fracture accompanied by a radial nerve palsy prior to an


attempt at reduction
2- A fracture with more than 30 degrees of varus angulation prior to reduction
3- A distal one third spiral fracture
4- A fracture caused by a low-velocity hand gun treated initially with wound
debridement and antibiotics
5- An inability to maintain less than 30 degrees of varus and 20 degrees of anterior
or posterior angulation after reduction

PREFERRED RESPONSE: 5

Dr. Ahmed Altaei


American Academy of Orthopaedic Surgeons
2002 shoulder Self-Assessment Examination

DISCUSSION: Most closed humeral shaft fractures and fractures caused by a low-velocity hand
gun can be managed nonsurgically with closed reduction and application of a coaptation splint
followed by a functional brace. Contraindications to use of the functional brace include:
1) massive soft-tissue or bone loss; 2) an unreliable or noncompliant patient; and 3) an inability
to maintain acceptable fracture alignment of up to 20 degrees of anterior or posterior angulation, Page | 35
30 degrees of varus or valgus angulation, and greater than 3 cm of shortening.

 Beaty JH (ed): Orthopaedic Knowledge Update 6. Rosemont, IL, American Academy of Orthopaedic Surgeons, 1999, pp
271-286.
 Pollock FH, Drake D, Bovill EG, Day L, Trafton PG: Treatment of radial neuropathy associated with fractures of the
humerus. J Bone Joint Surg Am 1981;63:239-243.
 Sarmiento A. Zagorski JB, Zych GA, et al: Functional bracing for the treatment of fractures of the humeral diaphysis. J
Bone Joint Surg Am 2000;82:478-486.

50. A 20-year-old man with fascioscapulohumeral dystrophy has severe scapular


winging of both shoulders. He can no longer abduct above 80 degrees, and it affects
his activities of daily living. A clinical photograph is shown in Figure 26. Definitive
management should consist of

1- a rehabilitation program to
strengthen his remaining
scapular muscles.
2- a scapular brace to keep his
scapula reduced.
3- scapulothoracic fusion.
4- pectoralis minor muscle transfer.
5- latissimus dorsi muscle transfer.

PREFERRED RESPONSE: 3

DISCUSSION: The patient’s history is typical of patients with severe fascioscapulohumeral


dystrophy. The scapular winging can be so pronounced that there is significant loss of function
of the upper extremity. The surgical options include transfer of the pectoralis major muscle with
a tendon graft or scapulothoracic fusion. The latter is a technically demanding procedure but can
provide a very stable platform for the upper extremity. Most patients will see increased elevation
of the extremity once the scapula is stabilized. Pectoralis minor transfer has not been described
and would not be effective.

 Shapiro F, Specht L: The diagnosis and orthopaedic treatment of inherited muscular diseases of childhood. J Bone Joint
Surg Am 1993;75:439-454.
 Bunch WH, Siegel IM: Scapulothoracic arthrodesis in fascioscapulohumeral muscular dystrophy: Review of seventeen
procedures with three to twenty-one-year follow-up. J Bone Joint Surg Am 1993;75:372-376.
51. Flexion and extension of the elbow occur about an axis of rotation that

1- corresponds with a line drawn through the centers of the trochlea and the
capitellum.
2- corresponds with a line drawn through the center of the medial epicondyle and the
lateral epicondyle.
3- corresponds with a line drawn through the radial head and coronoid.
4- moves with flexion and extension.
5- is polycentric.

PREFERRED RESPONSE: 1

DISCUSSION: The elbow mimics a true hinge and flexes and extends around an axis that is
centered in the centers of the trochlea and capitellum. The medial epicondyle is not perfectly
isometrically placed; rather the axis of rotation passes through a point on the anteroinferior
aspect of the medial epicondyle. Application of a hinged external fixator is possible because of
the fact that there is a single axis of rotation.

 Morrey BF (ed): The Elbow and Its Disorders, ed 2. Philadelphia, PA, WB Saunders, 1993, pp 53-54.
 London JT: Kinematics of the elbow. J Bone Joint Surg Am 1981;63:529-535.
 Morrey BF, Chao EY: Passive motion of the elbow joint. J Bone Joint Surg Am 1976;58:501-508.

52. Figure 27 shows the radiograph of a 26-year-old man who sustained a closed head
injury and a closed elbow dislocation 6 weeks ago. Examination reveals 65 degrees
to 115 degrees of flexion, and intensive physical therapy has resulted in no
improvement. A decision regarding the timing of surgical correction of the
contracture should be based on

1- bone scan results returning to


normal.
2- a decline in intensity on serial
bone scans.
3- the serum levels of alkaline
phosphatase measured over time.
4- the level of serum calcium-
phosphorus product.
5- the time since injury and
evidence of bone maturation on
plain radiographs.

PREFERRED RESPONSE: 5

DISCUSSION: The patient has heterotopic ossification, a more common finding in patients who
have sustained head injuries. Treatment will require removal of the heterotopic bone and
anterior and posterior capsulectomies. The main concern about timing is the possible recurrence

Dr. Ahmed Altaei


American Academy of Orthopaedic Surgeons
2002 shoulder Self-Assessment Examination

of heterotopic bone. While an extended wait was once thought necessary, this is no longer true.
The timing is based on the time since injury and evidence of bone maturation on plain
radiographs. A sharp marginal demarcation of the new bone and a trabecular pattern within it
are usually present 3 to 6 months after onset, indicating that it is safe to proceed with surgical
excision. It is not necessary to wait more than 6 months. Bone scan results are not good
indicators because they may remain “hot” for long periods of time. The levels of alkaline Page | 37
phosphatase and serum calcium-phosphorus product do not need to be measured.

 Norris TR (ed): Orthopaedic Knowledge Update: Shoulder and Elbow. Rosemont, IL, American Academy of Orthopaedic
Surgeons, 1997, pp 325-335.

53. A 70-year-old man who underwent an uncomplicated large rotator cuff repair 6
months ago is now seeking a second opinion regarding persistent pain and weakness
in his shoulder. Examination reveals that his incision is well healed and unreactive.
The surgical report suggests that the tendons were secured back to bone with
sutures through the greater tuberosity. Figure 28 shows a radiograph that was
obtained 1 week ago. What is the most likely diagnosis?

1- Infection
2- Complex regional pain
syndrome with
associated osteopenia
3- Frozen shoulder
4- Failed rotator cuff repair
5- Acromioclavicular joint
arthritis

PREFERRED RESPONSE: 4

DISCUSSION: Symptoms can persist following a rotator cuff repair for a variety of reasons. In
the early postoperative period, infection is the primary concern. Stiffness and loss of motion can
occur because of postoperative scarring. Complex regional pain syndrome can occur but is rare,
and the diagnosis is not made with a plain radiograph. This radiograph shows a superiorly
migrated humeral head that articulates with the acromion, indicating that the repair has failed.
While large to massive tears may fail more commonly than once thought, the clinical outcome
may be satisfactory in many patients.

 Mansat P, Cofield RH, Kersten TE, Rowland CM: Complications of rotator cuff repair. Orthop Clin North Am
1997;28:205-213.
 Jost B, Pfirrmann CW, Gerber C, Switzerland Z: Clinical outcome after structural failure of rotator cuff repairs. J Bone
Joint Surg Am 2000;82:304-314.
54. A 29-year-old man who lifts weights states that he injured his left shoulder while
performing a bench press 2 days ago. The following morning he noted ecchymosis
and swelling in the left chest wall. Examination reveals ecchymosis and tenderness
and deformity in the left anterior chest wall and axillary fold that is accentuated
with resisted adduction of the arm. Passive range of motion beyond 90 degrees of
forward flexion and 45 degrees of external rotation is extremely painful.
Glenohumeral stability is difficult to assess because of severe guarding. Figure 29
shows an MRI scan. Management should consist of

1- proximal biceps tenodesis.


2- application of a sling for 3 weeks, followed by physical therapy.
3- anterior capsulolabral reconstruction.
4- repair of the subscapularis tendon.
5- repair of the pectoralis major tendon.

PREFERRED RESPONSE: 5

DISCUSSION: Rupture of the pectoralis major tendon most commonly occurs during bench
pressing. Wolfe and associates have shown that the most inferiorly located fibers of the sternal
head lengthen disproportionately during the final 30 degrees of humeral extension during the
bench press. This creates a mechanical disadvantage in the final portion of the eccentric phase of
the lift; with forceful flexion of the shoulder these maximally stretched fibers may rupture. In
most patients, particularly in young athletes, the treatment of choice is anatomic repair of the
ruptured tendon to its insertion in the proximal humerus either with suture anchors or
transosseous sutures. Following surgery, most patients experience a near normal return of
strength and significant improvement in the cosmetic appearance of the deformity. While more
technically challenging, repair of chronic rupture is possible and is indicated in some patients.

Dr. Ahmed Altaei


American Academy of Orthopaedic Surgeons
2002 shoulder Self-Assessment Examination

 Wolfe SW, Wickiewicz TL, Cavanaugh JT: Ruptures of the pectoralis major muscle: An anatomic and clinical analysis.
Am J Sports Med 1992;20:587-593.
 Schepsis AA, Grafe MW, Jones HP, Lemos MJ: Rupture of the pectoralis major muscle: Outcome after repair of acute and
chronic injuries. Am J Sports Med 2000;28:9-15.

Page | 39
55. What range of motion parameters are required for a patient with posttraumatic
elbow stiffness to accomplish all the normal activities of daily living?

1- Flexion and extension of 10 degrees to 110 degrees, pronation of 50 degrees, and


supination of 50 degrees
2- Flexion and extension of 10 degrees to 130 degrees, pronation of 50 degrees, and
supination of 50 degrees
3- Flexion and extension of 30 degrees to 110 degrees, pronation of 60 degrees, and
supination of 30 degrees
4- Flexion and extension of 30 degrees to 130 degrees, pronation of 50 degrees, and
supination of 50 degrees
5- Flexion and extension of 30 degrees to 130 degrees, pronation of 60 degrees, and
supination of 30 degrees

PREFERRED RESPONSE: 4

DISCUSSION: Activities of daily living such as dressing, eating, and bathing can all be
performed with elbow motion through a 100 degrees arc of flexion and extension (30 degrees to
130 degrees) and a 100 degrees arc of forearm rotation (50 degrees pronation, 50 degrees
supination). Some patients can accomplish these activities of daily living with 10 degrees less
motion at each end point. This is referred to as the functional arc of motion.

 Kasser JR (ed): Orthopaedic Knowledge Update 5. Rosemont, IL, American Academy of Orthopaedic Surgeons, 1996, pp
283-294.
 Morrey BF, Askew LJ, Chao EY: A biomechanical study of normal functional elbow motion. J Bone Joint Surg Am
1981;63:872-877.

56. A 24-year-old athlete has a painful


right shoulder. Figure 30 shows an
intra-articular photograph that
was obtained through a posterior
portal during arthroscopy; the
labrum is indicated by the arrow.
Based on these findings,
management should consist of

1- stabilization with suture


anchors.
2- debridement only.
3- no treatment.
4- stabilization using absorbable tacks with the arm in external rotation.
5- release of the attachment to the middle glenohumeral ligament, followed by
stabilization with any device.

PREFERRED RESPONSE: 3

DISCUSSION: The photograph shows a normal variant that is a sublabral hole beneath the
anterosuperior labrum. In some instances, the labrum will become confluent with the middle
glenohumeral ligament as a stout band. Because this variant is not abnormal, no treatment is
necessary. Securing this portion of the labrum to the capsule may tighten the middle
glenohumeral ligament complex and restrict external rotation of the arm.

 Andrews JR, Guerra JJ, Fox GM: Normal and pathologic arthroscopic anatomy of the shoulder, in Andrews JR, Timmerman
LA (eds): Diagnostic and Operative Arthroscopy, ed 1. Philadelphia, PA, WB Saunders, 1997, pp 60-76.
 Williams MM, Snyder SJ, Buford D Jr: The Buford complex: The “cord-like” middle glenohumeral ligament and absent
anterosuperior labrum complex. A normal anatomic capsulolabral variant. Arthroscopy 1994;10:241-247.

57. The use of a screw between the clavicle and the coracoid process to maintain the
clavicle and acromioclavicular (AC) joint in a reduced position is a treatment option
for AC joint separations. Screw removal is generally recommended after soft-tissue
healing. What effect does this rigid coracoclavicular fixation have on shoulder
kinematics?

1- Significant limitation of humeral elevation


2- Significant limitation of shoulder abduction
3- Significant loss of motion in all directions
4- Little to no limitation of shoulder range of motion
5- Limitation of humeral rotation

PREFERRED RESPONSE: 4

DISCUSSION: This issue has been debated since Inman published his classic study on clavicular
rotation in 1944. Subsequently, it has been shown by several authors that the clinical evaluation
of patients with either coracoclavicular screws in place or with arthrodesis of the
coracoclavicular reveals little to no loss of shoulder motion. This is most likely the result of
synchronous motion of the scapula and clavicle in shoulder movements.

 Flatow EL: The biomechanics of the acromioclavicular, sternoclavicular, and scapulothoracic joints. Instr Course Lect
1993;42:237-245.
 Kenedy JC, Cameron H: Complete dislocation of the acromioclavicular joint. J Bone Joint Surg Br 1954;36:202-208.
 Rockwood CA Jr, Williams GR, Young CD: Disorders of the acromioclavicular joint, in Rockwood CA Jr, Matsen FA III
(eds): The Shoulder. Philadelphia, PA, WB Saunders, 1998, vol 1, pp 483-553.
 Inman VT, Saunders JB, Abbott LC: Observations of the function of the shoulder joint. Clin Orthop 1996;330:3-12.

58. Figure 31 shows the AP and lateral radiographs of the elbow of a 56-year-old man
with chronic polyarticular rheumatoid arthritis. His function continues to be

Dr. Ahmed Altaei


American Academy of Orthopaedic Surgeons
2002 shoulder Self-Assessment Examination

limited by pain with activities of daily living. Examination shows that his total arc
of motion is 110 degrees. Nonsurgical management has failed to provide relief.
Treatment should now consist of

Page | 41

1- elbow fusion with a contoured dynamic compression plate.


2- radial head excision and synovectomy.
3- distraction arthroplasty with interpositional tissue.
4- total elbow replacement with an unconstrained prosthesis.
5- total elbow replacement with a semiconstrained prosthesis.

PREFERRED RESPONSE: 5

DISCUSSION: A semiconstrained prosthesis can provide excellent results in carefully selected


patients. Because the radiographs show extensive joint destruction with loss of the capitellum
and trochlea, a capitellocondylar total elbow (unconstrained) prosthesis is contraindicated.
Elbow fusion is poorly accepted, and the radiographs show too much articular destruction for a
radial head excision, synovectomy, or interposition arthroplasty to be effective.

 Ewald FC, Simmons ED Jr, Sullivan JA, et al: Capitellocondylar total elbow replacement in rheumatoid arthritis: Long-term
results. J Bone Joint Surg Am 1993;75:498-507.
 Morrey BF, Adams RA: Capitellocondylar total elbow replacement in rheumatoid arthritis. J Bone Joint Surg Am
1992;74:479-490.

59. A 12-year-old pitcher has had a 2-month history of pain in his right dominant
shoulder after throwing. He reports that the pain has gradually progressed to the
point where he cannot throw without pain. He also notes that the pain now awakens
him at night if he has been active. Anti-inflammatory drugs have failed to provide
relief. Examination reveals no abnormalities except for some localized tenderness
over the proximal humerus. Figures 32a and 32b show radiographs of both
shoulders. What is the most likely diagnosis?
1- Chondroblastoma
2- Osteoid osteoma
3- Occult instability
4- Rotator cuff tear
5- Injury to the proximal
humeral physis

PREFERRED RESPONSE: 5

DISCUSSION: The history, examination, and radiographs are pathognomonic for Little League
shoulder, a stress syndrome of the proximal humeral physis caused by overuse. Complete
fracture rarely occurs, and recovery usually occurs with rest. Night pain is always a serious
concern and further work-up is needed if the patient does not respond to activity modification.
Occult instability is not a real concern in this patient, although it should be included in the
differential diagnosis.

 Albert MJ, Drvaric DM: Little League shoulder: Case report. Orthopedics 1990;13:779-781.
 Barnett LS: Little League shoulder syndrome: Proximal humeral epiphyseolysis in adolescent baseball pitchers. A case
report. J Bone Joint Surg Am 1985;67:495-496.

60. Which of the following ligaments is the primary static restraint against inferior
translation of the arm when the shoulder is in 0 degrees of abduction?

1- Middle glenohumeral
2- Inferior glenohumeral
3- Coracoacromial
4- Coracoclavicular
5- Coracohumeral

PREFERRED RESPONSE: 5

DISCUSSION: The superior glenohumeral ligament (SGHL) and coracohumeral ligament serve
as primary static restraints against inferior translation of the arm when the shoulder is in 0
degrees of abduction. Of these, the coracohumeral ligament has been shown to have a greater
cross-sectional area, greater stiffness, and greater ultimate load than the SGHL. The inferior
glenohumeral ligament plays a greater stabilizing role with increasing abduction of the arm. The
coracoacromial ligament may help provide superior stability, especially when the rotator cuff is
deficient. The coracoclavicular ligaments stabilize the acromioclavicular joint.

Dr. Ahmed Altaei


American Academy of Orthopaedic Surgeons
2002 shoulder Self-Assessment Examination

 Boardman ND, Debski RE, Warner JJ, et al: Tensile properties of the superior glenohumeral and coracohumeral ligaments.
J Shoulder Elbow Surg 1996;5:249-254.
 Warner JJ, Deng XH, Warren RF, Torzilli PA: Static capsuloligamentous restraints to superior-inferior translation of the
glenohumeral joint. Am J Sports Med 1992;20:675-685.

Page | 43
61. A 16-year-old high school student undergoes a routine preparticipation physical
examination at the beginning of the school year. Examination reveals marked laxity
of both shoulders but only mild generalized laxity in other joints. The load and shift
test allows for anterior humeral translation to the glenoid rim and posterior
humeral translation beyond the glenoid rim. The sulcus sign is present. What is the
next most appropriate step in management?

1- Inform the student that participation in sports is prohibited.


2- Order MRI of the shoulders to evaluate for labral tears.
3- Consider arthroscopic thermal capsulorraphy to tighten the shoulders.
4- Consider open capsular shift procedures to stabilize the shoulders.
5- Recommend a program of shoulder strengthening exercises and allow
participation in sports.

PREFERRED RESPONSE: 5

DISCUSSION: This patient has shoulder laxity without apprehension. Because there is a wide
range of normal laxity in asymptomatic shoulders, the physician should inform the student of
these findings, recommend shoulder strengthening exercises, and allow unrestricted sports
participation unless symptoms develop.

 Harryman DT, Sidles JA, Harris SL, Matsen FA III: Laxity of the normal glenohumeral joint: A quantitative in vivo
assessment. J Shoulder Elbow Surg 1992;1:66-76.
 Hawkins RJ, Bokor RJ: Clinical evaluation of shoulder problems, in Rockwood CA Jr, Matsen FA III (eds): The Shoulder.
Philadelphia, PA, WB Saunders, 1998, vol 1, p 186.
 McFarland EG, Campbell G, McDowell J: Posterior shoulder laxity in asymptomatic athletes. Am J Sports Med
1996;24:468-471.
 Linter SA, Levy A, Kenter K, Speer KP: Glenohumeral translation in the asymptomatic athlete’s shoulder and its
relationship to clinically measurable anthropometric variables. Am J Sports Med 1996;24:716-720.

62. A 21-year-old professional baseball player has had painful catching and stiffness in
his dominant right elbow for the past year. Examination reveals a flexion
contracture of 2 degrees and mild pain with full elbow flexion. Radiographs are
shown in Figures 33a and 33b. The most effective management should consist of
1- reconstruction of the medial collateral ligament.
2- a short period of rest followed by a gradual return
to activity.
3- physical therapy and dynamic extension splinting.
4- arthroscopic removal of the loose body.
5- a corticosteroid injection.

PREFERRED RESPONSE: 4

DISCUSSION: The radiographs show osteochondritis dissecans of the capitellum and a loose
body in the anterior compartment. Arthroscopic removal is indicated because symptoms
referable to the loose body are present.

 Baumgarten TE: Osteochondritis dissecans of the capitellum. Sports Med Arthroscopy Rev 1995;3:219-223.
 Shaughnessy WJ, Bianco AJ: Osteochondritis dissecans, in Morrey BF (ed): The Elbow and Its Disorders, ed 2.
Philadelphia, PA, WB Saunders, 1993, pp 282-287.

63. A 42-year-old patient has had painful inferior


subluxation of the glenohumeral joint
following a recent cerebrovascular accident
(CVA). Figure 34 shows the AP radiograph of
the shoulder. Management should consist of

1- closed reduction.
2- symptomatic sling support and range-of-
motion exercises.
3- arthroscopic thermal capsulorrhaphy.
4- an open anterior-inferior capsular shift.
5- a Laterjet procedure.

PREFERRED RESPONSE: 2

Dr. Ahmed Altaei


American Academy of Orthopaedic Surgeons
2002 shoulder Self-Assessment Examination

DISCUSSION: Following a CVA and with the resumption of upright posture, downward
subluxation of the glenohumeral joint may occur. Although usually painless, some patients may
report pain secondary to stretching of the brachial plexus. This is the result of flaccid paralysis
of the deltoid muscle, and it will persist until some motor tone or spasticity returns to the
shoulder girdle musculature. Early sling support and range-of-motion exercises to prevent
contracture will provide the best relief. Surgical procedures are not indicated. Page | 45

 Braun RM, Botte MJ: Treatment of shoulder deformity in acquired spasticity. Clin Orthop 1999;368:54-65.
 McCollough NC III: Orthopaedic evaluation and treatment of the stroke patient. Instr Course Lect 1975;24:45-55.

64. A 50-year-old man who underwent an arthroscopic rotator cuff repair 5 days ago
now returns for an early postoperative follow-up because of increasing pain in his
shoulder. He reports increasing malaise and has a low-grade fever. Examination
reveals no redness or swelling, but he has scant serous drainage from the posterior
portal. An emergent Gram stain is positive for gram-positive cocci. The next most
appropriate step in management should consist of

1- oral antibiotics and observation.


2- IV antibiotics and observation.
3- immediate arthroscopic debridement and lavage.
4- blood cultures, oral antibiotics, and a reculture in 2 days.
5- aspiration of the joint at his regular follow-up in 7 days if the symptoms increase.

PREFERRED RESPONSE: 3

DISCUSSION: An infection of the shoulder is considered a surgical emergency unless there are
medical reasons that a patient cannot be taken to the operating room. If cultures of wound
drainage are in question, then an aspiration should be done emergently, not several days later.
The hallmark of infection in any major joint is increasing pain out of proportion to what is
expected. Drainage occurring 1 to 2 days after an arthroscopic procedure is not normal, and it
should be aggressively treated. Delay in diagnosis can result in sepsis and on a delayed basis,
postinfectious arthritis. Both the glenohumeral joint and the subacromial space require
debridement and irrigation, followed by antibiotics after both areas are cultured.

 Mansat P, Cofield RH, Kersten TE, Rowland CM: Complications of rotator cuff repair. Orthop Clin North Am
1997;28:205-213.
 Settecerri JJ, Pitner MA, Rock MG, Hanssen AD, Cofield RH: Infection after rotator cuff repair. J Shoulder Elbow Surg
1999;8:1-5.
 Ward WG, Eckardt JJ: Subacromial/subdeltoid bursa abscesses: An overlooked diagnosis. Clin Orthop 1993;288:189-194.
 Ward WG, Goldner RD: Shoulder pyarthrosis: A concomittant process. Orthopedics 1994;17:591-595.

65. A 42-year-old man who is right-hand dominant injured his right shoulder when he
fell from a ladder onto his outstretched arm 1 hour ago. Radiographs reveal a two-
part greater tuberosity anterior fracture-dislocation. Initial management should
consist of
1- closed reduction of the glenohumeral joint and open reduction of the displaced
greater tuberosity with rotator cuff repair.
2- closed reduction of the glenohumeral joint, followed by radiographic assessment
of the tuberosity position to determine further treatment.
3- open reduction of both the joint and greater tuberosity with rotator cuff repair.
4- open reduction of the glenohumeral joint and closed treatment of the greater
tuberosity.
5- use of a sling until the patient reports no discomfort, then early passive range of
motion.

PREFERRED RESPONSE: 2

DISCUSSION: Greater tuberosity anterior fractures associated with anterior glenohumeral


dislocations respond very well to closed methods in the majority of patients. Closed reduction of
the glenohumeral joint often anatomically reduces the greater tuberosity into its cancellous bed,
without the need for open fixation or cuff repair. Once closed reduction of the joint is
performed, tuberosity displacement and joint articulation should be evaluated radiographically
with AP and scapular lateral views as well as an axillary view. The axillary view will not only
definitively show the joint articulation but also demonstrate posterior displacement of the greater
tuberosity missed on the AP and lateral views. If no or minimal (5 mm) displacement is found,
then nonsurgical management consisting of a sling and gentle passive range-of-motion exercises
can be instituted.

 Beaty JH (ed): Orthopaedic Knowledge Update 6. Rosemont, IL, American Academy of Orthopaedic Surgeons, 1999, pp
271-286.
 Flatow EL, Cuomo F, Maday MG, Miller SR, McIlveen SJ, Bigliani LU: Open reduction and internal fixation of two-part
displaced fractures of the greater tuberosity of the proximal part of the humerus. J Bone Joint Surg Am 1991;73:1213-1218.

66. A 19-year-old man who plays college volleyball


undergoes a routine preparticipation physical
examination. Figure 35 shows a posterior view
of his dominant shoulder. An electromyogram
shows that this is a chronic injury, and an MRI
scan shows no abnormalities. The best course
of action should be

1- a program of shoulder strengthening exercises.


2- decompression of the nerve at the spinoglenoid
notch.
3- decompression of the nerve at the transverse
suprascapular ligament.
4- release of the fascial elements of the muscle
tethering the nerve.
5- arthroscopy, repair of the posterior labrum lesion,
and an anterior capsular shift.

Dr. Ahmed Altaei


American Academy of Orthopaedic Surgeons
2002 shoulder Self-Assessment Examination

PREFERRED RESPONSE: 1

DISCUSSION: Isolated palsy of the infraspinatus portion of the suprascapular nerve is common
in volleyball players and is seen frequently in the throwing arm of baseball players. The exact
cause is not known, but it may be the result of either tethering or traction on the nerve at the
spinoglenoid notch. Synovial cysts in the spinoglenoid notch also can be a cause, but the Page | 47
patient’s negative MRI findings rule out that entity. Because many isolated nerve palsies of the
infraspinatus branch are asymptomatic, initial management should always be nonsurgical.
Surprisingly, many athletes with this injury can participate fully in sports. Surgical treatment
with decompression at the notch is unpredictable and generally is indicated only if nonsurgical
management fails.

 Ferretti A, Cerullo G, Russo G: Suprascapular neuropathy in volleyball players. J Bone Joint Surg Am 1987;69:260-263.
 Martin SD, Warren RF, Martin TL, Kennedy K, O’Brien SJ, Wickiewicz TL: Suprascapular neuropathy: Results of non-
operative treatment. J Bone Joint Surg Am 1997;79:1159-1165.

67. A 59-year-old construction worker who is right-hand dominant has had right
shoulder pain for the past 9 months with no history of injury. Nonsurgical
management consisting of two cortisone injections, physical therapy for 3 months,
and nonsteroidal anti-inflammatory drugs has failed to provide lasting relief.
Examination reveals tenderness over the acromioclavicular (AC) joint and over the
subacromial bursa. He has positive Neer and Hawkins impingement signs and AC
joint pain with adduction of the shoulder. Radiographs are shown in Figures 36a
and 36b. An MRI scan reveals an intact rotator cuff. Management should now
consist of

1- open anterior acromioplasty and rotator cuff repair.


2- arthroscopic acromioplasty.
3- anterior acromioplasty and distal clavicle excision.
4- an open Mumford procedure.
5- immobilization in a sling for 4 weeks followed by additional physical therapy.

PREFERRED RESPONSE: 3
DISCUSSION: Because the patient has clinical and radiographic signs of AC arthritis and
subacromial impingement, the treatment of choice is anterior acromioplasty and distal clavicle
excision. Arthroscopic acromioplasty alone would not address the AC arthritis. The rotator cuff
is intact; therefore, rotator cuff repair is not indicated. An open Mumford procedure would
address the AC arthritis only and not the impingement symptoms. Immobilization might lead to
stiffness of the shoulder and is not recommended for treating impingement.

 Peterson CA, Altchek DW, Warren RF: Shoulder arthroscopy, in Rockwood CA Jr, Matsen FA III (eds): The Shoulder.
Philadelphia, PA, WB Saunders, 1990, pp 290-335.

68. What three structures are considered the primary constraints necessary for
elbow stability?

1- Coronoid, ulnar part of the lateral collateral ligament, capsule


2- Capsule, anterior band of the medial collateral ligament, radial head
3- Radial head, ulnar part of the lateral collateral ligament, capsule
4- Anterior band of the medial collateral ligament, coronoid, radial head
5- Ulnar part of the lateral collateral ligament, anterior band of the medial collateral
ligament, coronoid

PREFERRED RESPONSE: 5

DISCUSSION: The three primary constraints necessary for elbow stability in all directions are
the ulnar part of the lateral collateral ligament (also called the lateral ulnar collateral ligament),
the anterior band of the medial collateral ligament, and the coronoid. The radial head and
capsule are secondary constraints to elbow instability.

 Kasser JR (ed): Orthopaedic Knowledge Update 5. Rosemont, IL, American Academy of Orthopaedic Surgeons, 1996, pp
283-294.
 Norris TR (ed): Orthopaedic Knowledge Update: Shoulder and Elbow. Rosemont, IL, American Academy of Orthopaedic
Surgeons, 1997, pp 345-354.

69. A 68-year-old woman has been


progressing slowly after undergoing
humeral head replacement for a
four-part fracture 3 months ago. She
has not regained active elevation, she
feels an audible clunk on attempting
elevation, and she reports pain and
weakness. She used a sling for 2
weeks in the immediate postoperative
period. Radiographs are shown in
Figure 37a through 37c.
Management should consist of

Dr. Ahmed Altaei


American Academy of Orthopaedic Surgeons
2002 shoulder Self-Assessment Examination

Page | 49

1- tuberosity and rotator cuff repair with bone graft.


2- revision arthroplasty leaving the prosthesis proud to increase humeral length and
muscle tension.
3- revision total shoulder arthroplasty to neutralize eccentric glenoid wear.
4- revision of the humeral head replacement alone with increased retroversion.
5- additional therapy to include internal and external rotation strengthening of the
rotator cuff.

PREFERRED RESPONSE: 1

DISCUSSION: Immediate repair of the tuberosity and rotator cuff is recommended on


identifying the avulsion or nonunion. Revising the humeral component to increase tension and
length will overtighten the cuff and increase the chance of tuberosity pull-off. The glenoid is
uninvolved and should not be replaced. Attempts to strengthen the rotator cuff will be
unsuccessful because the insertions are no longer attached to the humerus when the tuberosities
avulse.

 Brown TD, Bigliani LU: Complications with humeral head replacement. Orthop Clin North Am 2000;31:77-90.
 Muldoon MP, Cofield RH: Complications of humeral head replacement for proximal humeral fractures. Instr Course Lect
1997;46:15-24.

70. What is the most important feature in choosing an outcome instrument to assess
shoulder disorders?

1- Ease of use
2- Validity
3- Ability to use it by mail or phone so the subject is not required to return in person
to measure the outcome
4- Inclusion of radiographic assessment at follow-up
5- Scoring that is on a 100-point scale so that it can be compared with other
instruments

PREFERRED RESPONSE: 2

DISCUSSION: There has been a recent increase in the use of outcome instruments to document
and measure effects of treatment of medical conditions, including shoulder disorders. The most
important feature of an instrument is whether it actually measures what it purports to measure;
this is defined as its validity.

 Leggin BG, Iannotti JP: Shoulder outcome measurement, in Iannotti JP, Williams GR (eds): Disorders of the Shoulder:
Diagnosis and Management. Philadelphia, PA, Lippincott Williams and Wilkins, 1999, p 1027.
 Norris TR (ed): Orthopaedic Knowledge Update: Shoulder and Elbow. Rosemont, IL, American Academy of Orthopaedic
Surgeons, 1997, pp 47-55.

71. Figure 38 shows the radiograph of a 16-year-old wrestler who injured his elbow
when he was thrown to the mat by his opponent. To minimize additional trauma to
the medial soft tissues, the elbow should be reduced in

1- neutral rotation.
2- full pronation.
3- full supination.
4- full extension.
5- full flexion.

PREFERRED RESPONSE: 3

DISCUSSION: The elbow dislocates by a three-dimensional movement of supination and valgus


during flexion. Additional trauma during reduction is minimized by recreating the deformity and
reducing the elbow in supination. The actual maneuver includes full supination (actually
hypersupination) of the elbow in a valgus position. This is followed by pushing the olecranon
distally in line with the long axis of the ulna while swinging the elbow into varus, and then
relaxing the supination torque. Postreduction stability is enhanced in pronation, except when the
soft-tissue disruption is extensive.

 O’Driscoll SW: Elbow dislocations, in Morrey BF (ed): The Elbow and Its Disorders, ed 2. Philadelphia, PA, WB
Saunders, 1993, p 414.
 Norris TR (ed): Orthopaedic Knowledge Update: Shoulder and Elbow. Rosemont, IL, American Academy of Orthopaedic
Surgeons, 1997, pp 345-354.

Dr. Ahmed Altaei


American Academy of Orthopaedic Surgeons
2002 shoulder Self-Assessment Examination

72. In patients older than age 40 years who sustain a first-time anterior dislocation of
the shoulder, prolonged morbidity is most commonly associated with

1- recurrent dislocation.
2- posttraumatic arthritis.
3- a rotator cuff tear. Page | 51
4- stiffness secondary to immobilization.
5- nerve injury.

PREFERRED RESPONSE: 3

DISCUSSION: In the study done by Pevny and associates, 35% of patients older than age 40
years sustained rotator cuff tears and 8% had axillary nerve palsies. All of the patients with
axillary nerve palsy also had rotator cuff tears. Imaging of the rotator cuff is indicated in this age
group. The incidence of recurrent instability in patients older than age 40 years is 10% to 15%.

 Pevny T, Hunter RE, Freeman JR: Primary traumatic anterior shoulder dislocation in patients 40 years of age and older.
Arthroscopy 1998;14:289-294.
 Sonnabend DH: Treatment of primary anterior shoulder dislocation in patients older than 40 years of age: Conservative
versus operative. Clin Orthop 1994;304:74-77.
 Hawkins RJ, Mohtadi NG: Controversy in anterior shoulder instability. Clin Orthop 1991;272:152-161.

73. Figure 39 shows the AP radiograph of a 62-year-old man with degenerative


osteoarthritis secondary to trauma. History reveals that he underwent total elbow
arthroplasty 3 years ago. He continues to report instability and constant pain. A
complete work-up, including aspiration and cultures, is negative. Treatment should
consist of removal of the components and

1- distraction interpositional arthroplasty.


2- elbow arthrodesis.
3- conversion to a resection arthroplasty.
4- conversion to semiconstrained elbow
arthroplasty.
5- revision to unconstrained total elbow
arthroplasty.

PREFERRED RESPONSE: 4

DISCUSSION: An unconstrained prosthesis dislocation is a disconcerting problem that is not


easily resolved; however, revision to a semiconstrained prosthesis would best achieve both pain
relief and stability. Removal of the components and distraction arthroplasty or conversion to a
resection arthroplasty are options, but the results would be unpredictable with regards to pain
relief, postoperative motion, or elbow stability. Arthrodesis is poorly tolerated. With revision to
another unconstrained prosthesis, there is the risk of continued redislocation because of chronic
ligamentous insufficiency.

 Linscheid RL: Resurfacing elbow replacement arthroplasty: Rationale, technique and results, in Morrey BF (ed): The
Elbow and Its Disorders, ed 3. Philadelphia, PA, WB Saunders, 2000, pp 602-610.
 Morrey BF, King GJ: Revision of failed total elbow arthroplasty, in Morrey BF (ed): The Elbow and Its Disorders, ed 3.
Philadelphia, PA, WB Saunders, 2000, pp 685-700.

74. A 67-year-old woman undergoes a revision total shoulder arthroplasty for


replacement of a loose glenoid component. Examination in the recovery room
reveals absent voluntary deltoid and triceps contraction, weakness of wrist and
thumb extension, and absent sensation in the palmar aspect of all fingertips and the
radial forearm. The next most appropriate step in management should consist of

1- an immediate return to the operating room to explore the brachial plexus.


2- immediate electromyography and nerve conduction velocity studies.
3- MRI of the brachial plexus.
4- MRI of the cervical spine.
5- immobilization in a sling, followed by early passive range of motion.

PREFERRED RESPONSE: 5

DISCUSSION: Neurologic injury after shoulder replacement is relatively uncommon, occurring


in 4% of shoulders in one large series. The importance of identifying and protecting the
musculocutaneous and axillary nerves cannot be overemphasized; it is especially critical during
revision arthroplasty when the normal anatomic relationships have been distorted. The long
deltopectoral approach leaving the deltoid attached to the clavicle was found to be significant in
the development of postoperative neurologic complications. A correlation was found between
surgical time and postoperative neurologic complications, with long surgical times being
associated with more neurologic complications. The presumed mechanism of injury is traction
on the plexus that occurs during the surgery. A neurologic injury after total shoulder arthroplasty
usually does not interfere with the long-term outcome of the arthroplasty itself; it is best
managed by protective measures with passive range of motion of the involved extremity.

 Wirth MA, Rockwood CA Jr: Complications of shoulder arthroplasty. Clin Orthop 1994;307:47-69.
 Lynch NM, Cofield RH, Silbert PL, Hermann RC: Neurologic complications after total shoulder arthroplasty. J Shoulder
Elbow Surg 1996;5:53-61.

75. Figure 40 shows the radiograph of a 16-year-old wrestler who injured his elbow
when he was thrown to the mat by his opponent. Closed reduction is readily
accomplished, and the elbow seems stable. Management should now consist of
application of a splint for

Dr. Ahmed Altaei


American Academy of Orthopaedic Surgeons
2002 shoulder Self-Assessment Examination

1- 2 to 5 days, followed by
initiation of assisted motion.
2- 14 to 21 days, followed by
initiation of assisted motion.
3- 4 weeks, followed by active Page | 53
motion.
4- 6 weeks, followed by
physical therapy.
5- 8 weeks, followed by active
motion of the elbow.

PREFERRED RESPONSE: 1

DISCUSSION: Flexion contractures are the most common complication of elbow dislocations.
About 15% of patients lose more than 30 degrees of flexion. The risk of contracture is
proportional to the duration of immobilization. Elbows should be moved within the first few
days after reduction. The splinting is for comfort and protection only while the pain subsides.

 Mehlhoff TL, Noble PC, Bennett JB, Tullos HS: Simple dislocation of the elbow in the adult: Results after closed treatment.
J Bone Joint Surg Am 1988;70:244-249.
 Linscheid RL, O’Driscoll SW: Elbow dislocations, in Morrey BF (ed): The Elbow and Its Disorders, ed 2. Philadelphia,
PA, WB Saunders, 1993, pp 441-452.
 O’Driscoll SW, Jupiter JB, King GJ, Hotchkiss RN, Morrey BF: The unstable elbow. Instr Course Lect 2001;50:89-102.
 Ross G, McDevitt ER, Chronister R, Ove PN: Treatment of simple elbow dislocation using an immediate motion protocol.
Am J Sports Med 1999;27:308-311.

76. A 50-year-old electrician who is right-hand dominant has had right shoulder pain
and stiffness after sustaining an electric shock 2 months ago. An AP radiograph
obtained at the time of injury was considered negative, and the patient was
diagnosed with a shoulder sprain. The patient now reports continued shoulder pain
and restricted motion. AP and axillary radiographs and a CT scan are shown in
Figures 41a through 41c. Management should consist of
1- continued observation and physical therapy.
2- closed reduction in the office.
3- closed reduction under anesthesia in the hospital.
4- humeral arthroplasty.
5- open reduction and transfer of the subscapularis and lesser tuberosity into the
anteromedial humeral head defect.

PREFERRED RESPONSE: 5

DISCUSSION: Open reduction and transfer of the subscapularis and lesser tuberosity into the
humeral head defect is the treatment of choice for chronic posterior dislocations in which the
articular defect consists of 20% to 40% of the articular surfaces. Closed reduction can be used if
the dislocation is recognized early and the articular defect is less than 20% of the articular
surface. Humeral arthroplasty is reserved for patients with an articular defect that is greater than
45% to 50% of the head.

 Hawkins RJ, Neer CS II, Pianta RM, Mendoza FX: Locked posterior dislocation of the shoulder. J Bone Joint Surg Am
1987;69:9-18.
 Checchia SL, Santos PD, Miyazaki AN: Surgical treatment of acute and chronic posterior fracture-dislocation of the
shoulder. J Shoulder Elbow Surg 1998;7:53-65.

77. Figure 42 shows the radiograph of a 70-year-old


woman who has had a painful near ankylosis of
her dominant elbow for 1 year. Treatment
should consist of

1- total elbow replacement.


2- hardware removal and joint release.
3- medial and lateral column humerus plating
and a bone graft.
4- distal humerus replacement.
5- resection arthroplasty.

Dr. Ahmed Altaei


American Academy of Orthopaedic Surgeons
2002 shoulder Self-Assessment Examination

PREFERRED RESPONSE: 1

DISCUSSION: The patient has arthritis and supracondylar nonunion of the elbow. Total elbow
replacement has been shown to give almost immediate return of function as it can be performed
while leaving the triceps intact and resecting the distal humerus fragment. Attempts at
osteosynthesis are indicated in younger individuals with good joint surface. Resection Page | 55
arthroplasty yields poor function and is reserved as a salvage procedure.

 Ramsey ML, Morrey BF: Total elbow arthroplasty for nonunion and dysfunctional instability, in Morrey BF (ed): The
Elbow and Its Disorders, ed 3. Philadelphia, PA, WB Saunders, 2000, pp 655-661.
 Sim FH, Morrey BF: Nonunion and delayed union of distal humeral fractures, in Morrey BF (ed): The Elbow and Its
Disorders, ed 3. Philadelphia, PA, WB Saunders, 2000, pp 655-661.

78. A 72-year-old woman who was doing well after undergoing total shoulder
arthroplasty for arthritis 4 months ago is suddenly unable to elevate her arm.
Examination reveals 70 degrees of external rotation compared with 45 degrees on
the uninvolved side, and she is unable to lift her hand off her lower back.
Radiographs are shown in Figures 43a through 43c. Treatment should consist of

1- fascia lata graft to restore the coracoacromial arch.


2- immediate subscapularis repair.
3- revision arthroplasty with glenoid reaming to centralize the component.
4- revision arthroplasty with increased retroversion in the humeral component.
5- arthroscopic subacromial decompression.

PREFERRED RESPONSE: 2

DISCUSSION: Results of treatment of subscapularis rupture are best when immediate repair is
performed. When the cause of the anterior instability is the result of rupture of the subscapularis
tendon and the component position is acceptable, revising the position of the component is
unnecessary. Restoring the coracoacromial arch and subacromial decompression are related to
superior instability and rotator cuff pathology, respectively, and would not correct the instability
caused by subscapularis rupture.
 Moeckel BH, Altchek DW, Warren RF, Wickiewicz TL, Dines DM: Instability of the shoulder after arthroplasty. J Bone
Joint Surg Am 1993;75:492-497.
 Gerber C, Hersche O, Farron A: Isolated rupture of the subscapularis tendon. J Bone Joint Surg Am 1996;78:1015-1023.

79. A 25-year-old man underwent a Putti-Platt repair for recurrent anterior dislocation
of his right shoulder 9 months ago. He reports no further episodes of instability but
continues to have severely restricted motion, with external rotation limited to less
than 0 degrees with the arm at the side. He has pain at the ends of range of motion
and restricted activities of daily living despite undergoing nearly 9 months of
physical therapy. Radiographs of the shoulder show no arthritic changes.
Management should now consist of

1- additional physical therapy for 6 months followed by reassessment.


2- manipulation under anesthesia.
3- arthroscopic release combined with the use of an interscalene catheter
postoperatively.
4- open release with Z-plasty lengthening of the subscapularis and capsule.
5- shoulder hemiarthroplasty.

PREFERRED RESPONSE: 4

DISCUSSION: Open release allows lengthening of the shortened subscapularis and is preferred
when there are extra-articular contractures. Arthroscopic release, combined with the use of an
interscalene catheter postoperatively, is an excellent treatment for capsular contractures but is
contraindicated after procedures that result in extracapsular shortening (ie, Magnuson-Stack,
Putti-Platt). Additional physical therapy or manipulation under anesthesia is not likely to be
helpful. Shoulder hemiarthroplasty is contraindicated with normal articular surfaces, but
prosthetic arthroplasty is sometimes necessary for arthritis associated with instability or overly
tight instability repairs.

 Harryman DT II, Matsen FA III, Sidles JA: Arthroscopic management of refractory shoulder stiffness. Arthroscopy
1997;13:133-147.
 Warner JJ: Frozen shoulder: Diagnosis and management. J Am Acad Orthop Surg 1997;5:130-140.
 Warner JJ, Allen AA, Marks PH, Wong P: Arthroscopic release of postoperative capsular contracture of the shoulder. J
Bone Joint Surg Am 1997;79:1151-1158.
 MacDonald PB, Hawkins RJ, Fowler PJ, Miniaci A: Release of the subscapularis for internal rotation contracture and pain
after anterior repair for recurrent anterior dislocation of the shoulder. J Bone Joint Surg Am 1992;74:734-737.

80. A 43-year-old bus driver sustains a hyperextension injury to her arm and shoulder 4
months after undergoing an open Bankart repair. Examination reveals increased
external rotation, anterior shoulder pain, and internal rotation weakness. Her
examination also reveals the findings shown in Figure 44. What is the most likely
diagnosis?

Dr. Ahmed Altaei


American Academy of Orthopaedic Surgeons
2002 shoulder Self-Assessment Examination

Page | 57

1- Superior labrum anterior and posterior lesion, type III


2- Isolated traumatic dislocation
3- Axillary nerve disruption
4- Subscapularis rupture
5- Internal impingement

PREFERRED RESPONSE: 4

DISCUSSION: An isolated tear of the subscapularis tendon has been noted as early as 1835 by
Smith. In Gerber and associates’ 1991 report of 16 men with an average age of 51 years, isolated
subscapularis tendon rupture was often caused by a violent hyperextension injury. All patients
reported pain anteriorly along with night pain. They also noted pain and weakness of the arm.
The lift-off test is performed by having the patient lift the palm of the hand away from the small
of the back. The patient must have sufficient internal rotation to allow this test to be performed.
A subscapularis rupture is likely if the patient cannot perform the lift-off test.

 Hertel R, Ballmer FT, Lombert SM, Gerber C: Lag signs in the diagnosis of rotator cuff rupture. J Shoulder Elbow Surg
1996;5:307-313.
 Gerber C, Krushell RJ: Isolated rupture of the tendon of the subscapularis muscle: Clinical features in 16 cases. J Bone
Joint Surg Br 1991;73:389-394.
 Greis PE, Kuhn JE, Schultheis J, Hintermeister R, Hawkins R: Validation of the lift-off test and analysis of subscapularis
activity during maximal internal rotation. Am J Sports Med 1996;24:589-593.
 Gerber C, Hersche O, Farron A: Isolated rupture of the subscapularis tendon. J Bone Joint Surg Am 1996;78:1015-1023.

81. Radial nerve palsy is most commonly associated with which of the following types of
humeral fractures?
1- Proximal one third spiral
2- Proximal one third transverse
3- Distal one third spiral
4- Distal one third transverse
5- Middle one third

PREFERRED RESPONSE: 5

DISCUSSION: Although the Holstein-Lewis fracture, described as an oblique distal one third
fracture, is best known for its association with neurologic injury, radial nerve palsy is most
commonly associated with middle one third humeral fractures. Most nerve injuries are
neurapraxias or axonotmeses, with up to 90% resolving in 3 to 4 months.

 Beaty JH (ed): Orthopaedic Knowledge Update 6. Rosemont, IL, American Academy of Orthopaedic Surgeons, 1999, pp
271-286.
 Pollock FH, Drake D, Bovill EG, Day L, Trafton PG: Treatment of radial neuropathy associated with fractures of the
humerus. J Bone Joint Surg Am 1981;63:239-243.

82. A 30-year-old firefighter sustained a longitudinal pulling injury to the arm while
attempting to move a heavy object during a fire. Figure 45 shows an MRI scan of
the elbow. Initial management should consist of

1- rest and a sling followed by a


gradual return to activities.
2- physical therapy and
extension-block bracing.
3- repair of the biceps tendon to
the brachialis muscle.
4- repair of the common flexor
origin.
5- anatomic repair of the distal
biceps tendon.

PREFERRED RESPONSE: 5

DISCUSSION: Because the MRI scan shows a complete rupture of the distal biceps tendon, the
preferred treatment is anatomic repair of the tendon to the radial tuberosity either with the use of
suture anchors or transosseous sutures through a two-incision technique. Several studies have
documented superior results with anatomic repair of the distal biceps tendon when compared
with results of nonsurgical management or repair of the tendon by attachment to the brachialis
muscle. Patients undergoing anatomic repair of the distal biceps tendon through a two-incision
technique typically regain a functional range of motion and nearly normal strength.

Dr. Ahmed Altaei


American Academy of Orthopaedic Surgeons
2002 shoulder Self-Assessment Examination

 D’Alessandro DF, Shields CL Jr, Tibone JE, Chandler RW: Repair of distal biceps tendon ruptures in athletes. Am J Sports
Med 1993;21:114-119.
 Boyd JB, Anderson LD: A method for reinsertion of the distal biceps brachii tendon. J Bone Joint Surg Am 1961;43:1041-
1043.
 Morrey BF, Askew LJ, An KN, Dobyns JH: Rupture of the distal tendon of the biceps brachii: A biomechanical study. J
Bone Joint Surg Am 1985;67:418-421.
 Failla JM, Amadio PC, Morrey BF, Beckenbaugh RD: Proximal radioulnar synostosis after repair of distal biceps brachii
rupture by the two-incision technique: Report of four cases. Clin Orthop 1990;253:133-136. Page | 59

83. Which of the following is considered a reasonable goal for arthroplasty surgery in
rotator cuff arthropathy?

1- Restore normal humeral head glenoid contact location


2- Restore full active overhead motion
3- Restore proper glenoid version with bone preparation and use of a cemented
glenoid component
4- Achieve formal decompression and acromioplasty with resection of the
coracoacromial ligament and distal clavicle
5- Achieve a secure closure of the subscapularis with an appropriate head size

PREFERRED RESPONSE: 5

DISCUSSION: Absence of the rotator cuff results in superior migration of the humeral head
because of unopposed deltoid function. This proximal migration results in eccentric loading of
glenoid components with early loosening. Hemiarthroplasty yields good pain relief with limited
goals of active elevation of 90 degrees. The coracoacromial arch should be preserved.
Achieving satisfactory subscapularis tension is preferred to the use of an oversized humeral
component.

 Zeman CA, Arcand MA, Cantrell JS, Skedros JG, Burkhead WZ Jr: The rotator cuff-deficient arthritic shoulder: Diagnosis
and surgical management. J Am Acad Orthop Surg 1998;6:337-348.
 Arntz CT, Jackins S, Matsen FA III: Prosthetic replacement of the shoulder for the treatment of defects in the rotator cuff
and the surface of the glenohumeral joint. J Bone Joint Surg Am 1993;75:485-491.
 Williams GR Jr, Rockwood CA Jr: Hemiarthroplasty in rotator cuff-deficient shoulders. J Shoulder Elbow Surg
1996;5:362-367.
 Zuckerman JD, Scott AJ, Gallagher MA: Hemiarthroplasty for cuff tear arthropathy. J Shoulder Elbow Surg 2000;9:169-
172.

84. What is the best surgical approach for the scapular


fracture shown in Figure 46?

1- Anterior
2- Anterior and superior
3- Posterior
4- Percutaneous pinning
5- Closed reduction
PREFERRED RESPONSE: 3

DISCUSSION: Indications for open reduction of glenoid intra-articular fractures include those
fractures with a 5-mm articular surface displacement or when the humeral head is subluxated
with the fracture fragment. Kavanaugh and associates and Leung and Lam have shown that the
posterior approach with plate fixation is best for most glenoid fractures, including the Ideberg
type II fracture shown here. The anterior approach is best used for anterior rim and transverse
fractures.

 Kavanagh BF, Bradway JK, Cofield RH: Open reduction and internal fixation of displaced intra-articular fractures of the
glenoid fossa. J Bone Joint Surg Am 1993;75:479-484.
 Leung KS, Lam TP: Open reduction and internal fixation of ipsilateral fractures of the scapular neck and clavicle. J Bone
Joint Surg Am 1993;75:1015-1018.
 Ideberg R: Unusual glenoid fractures: A report on 92 cases. Acta Orthop Scand 1995;66:395-397.

85. Management of a grade IV osteochondritis dissecans lesion of the capitellum should


consist of

1- use of a sling for 3 weeks followed by a gradual return to activities.


2- physical therapy.
3- arthroscopy with removal of the loose fragment.
4- arthroscopy with in situ drilling of the fragment.
5- internal fixation of the fragment.

PREFERRED RESPONSE: 3

DISCUSSION: Osteochondritis dissecans of the capitellum is seen most commonly in adolescent


athletes. It should be distinguished from osteochondrosis of the capitellum (Panner’s disease), a
self-limiting condition seen in younger patients.
Lesions are graded I through V based on radiographic and arthroscopic appearance. Grade I
lesions show intact but soft cartilage. Grade II lesions show fissuring of the overlying cartilage.
Grade III lesions show exposed bone or an attached osteoarticular flap that is not loose. Grade
IV lesions show a loose but nondisplaced osteoarticular flap. Grade V lesions show a displaced
fragment.
Simple excision of the loose osteoarticular flap is the treatment of choice for grade IV and V
lesions. More complex procedures such as drilling of the in situ lesion, bone grafting, or internal
fixation are associated with significantly worse results. While some authors advocate abrasion
chondroplasty, the long-term benefits of the procedure are yet to be proven.

 Baumgarten TE: Osteochondritis dissecans of the capitellum. Sports Med Arthroscopy Rev 1995;3:219-223.
 Shaughnessy WJ, Bianco AJ: Osteochondritis dissecans, in Morrey BF (ed): The Elbow and Its Disorders, ed 2.
Philadelphia, PA, WB Saunders, 1993, pp 282-287.

86. What preoperative factor correlates best with the outcome of rotator cuff repair?

Dr. Ahmed Altaei


American Academy of Orthopaedic Surgeons
2002 shoulder Self-Assessment Examination

1- Size of the tear


2- Age of the patient
3- Arm dominance
4- Rupture of the long head of the biceps
5- Preoperative pain score
Page | 61
PREFERRED RESPONSE: 1

DISCUSSION: The size of the rotator cuff tear in both anteroposterior and mediolateral
dimensions has been found to correlate best with outcome. Older patient age and rupture of the
long head of the biceps tend to be associated with larger tears and, therefore, may be associated
indirectly with a poorer outcome.

 Iannotti JP: Full-thickness rotator cuff tears: Factors affecting surgical outcome. J Am Acad Orthop Surg 1994;2:87-95.
 Iannotti JP, Bernot MP, Kuhlman JR, Kelley MJ, Williams GR: Postoperative assessment of shoulder function: A
prospective study of full-thickness rotator cuff tears. J Shoulder Elbow Surg 1996;5:449-457.

87. A 55-year-old woman with polyarticular rheumatoid arthritis has had progressively
increasing left shoulder pain for the past 2 years despite nonsurgical management.
No focal weakness is noted during examination of the shoulder. AP and axillary
radiographs are shown in Figures 47a and 47b. Treatment should consist of

1- arthroscopic synovectomy.
2- humeral arthroplasty.
3- unconstrained total shoulder arthroplasty.
4- constrained total shoulder arthroplasty with a fixed-fulcrum prosthesis.
5- glenohumeral arthrodesis.

PREFERRED RESPONSE: 3
DISCUSSION: Unconstrained total shoulder arthroplasty has been found to yield satisfactory
results in a high percentage of patients with rheumatoid involvement of the glenohumeral joint.
Pain relief has been more predictable with total shoulder arthroplasty than humeral arthroplasty,
and a glenoid component is favored when there is sufficient glenoid bone stock and an intact
rotator cuff. Constrained or fixed-fulcrum devices have an unacceptably high failure rate
because of loosening. Glenohumeral arthrodesis is avoided when the deltoid or rotator cuff is
functioning because the functional results after arthroplasty are superior when compared with
results of arthrodesis. Arthroscopic synovectomy may be helpful in early stages of the disease
before extensive cartilage damage has occurred.

 Boyd AD Jr, Thomas WH, Scott RD, Sledge CB, Thornhill TS: Total shoulder arthroplasty versus hemiarthroplasty:
Indications for glenoid resurfacing. J Arthroplasty 1990;5:329-336.
 Kelly IG, Foster RS, Fisher WD: Neer total shoulder replacement in rheumatoid arthritis. J Bone Joint Surg Br
1987;69:723-726.

88. When elevating the arm, the ratio of scapulothoracic to glenohumeral motion over the
total range of motion is best described as

1- 1:2, and in the first 30 degrees the ratio is 1:5.


2- 1:2, and in the first 30 degrees the ratio is variable.
3- 2:1, and in the first 30 degrees the ratio is variable.
4- 2:1, and in the first 30 degrees the ratio is 3:1.
5- highly variable and no definitive statement can be made about the ratios.

PREFERRED RESPONSE: 2

DISCUSSION: The ratio of scapulothoracic to glenohumeral motion with elevation has been
shown to vary depending on what portion of elevation is examined, how much load is on the
arm, and the technique used to measure increments of elevation. However, almost every study
shows that the ratio of scapulothoracic to glenohumeral motion is 1:2 for the contributions over a
full range of elevation to 170 degrees. In the first 30 degrees of elevation, there is significant
variability in the ratio, and there may be significant variability up to around 60 degrees.

 Inman VT, Saunders JR, Abbott LC: Observations of the function of the shoulder joint. Clin Orthop 1996;330:3-12.
 Freedman L, Munro RH: Abduction of the arm in the scapular plane: Scapular and glenohumeral movements. J Bone Joint
Surg Am 1966;18:1503.

89. Figure 48 shows the initial AP


chest radiograph of a 21-year-
old motorcycle rider who
sustained multiple injuries after
striking a telephone pole at high
speed. What is the most
significant radiographic finding
leading to a diagnosis?

Dr. Ahmed Altaei


American Academy of Orthopaedic Surgeons
2002 shoulder Self-Assessment Examination

1- Subdiaphragmatic free air


2- Right midshaft clavicular fracture
3- Right scapulothoracic dissociation
4- Left diaphragmatic rupture
5- Left sternoclavicular dislocation Page | 63

PREFERRED RESPONSE: 3

DISCUSSION: Scapulothoracic dissociation is a rare, violent traumatic injury in which the


scapula is torn away from the chest wall but the skin remains intact. Massive swelling and
ecchymosis are common. Neurovascular injury is the rule with possible subclavian or axillary
artery disruption and severe partial or complete brachial plexus paralysis. The diagnosis is made
on a nonrotated chest radiograph that shows significant lateral displacement of the medial
scapular border from the sternal notch. A right midshaft clavicular fracture is present but is not
considered the most significant finding.

 Ebraheim NA, An HS, Jackson WT, et al: Scapulothoracic dissociation. J Bone Joint Surg Am 1988;70:428-432.
 Ebraheim NA, Pearlstein SR, Savolaine ER, et al: Scapulothoracic dissociation. J Orthop Trauma 1987;1:18-23.
 Sampson LN, Britton JC, Eldrup-Jorgensen J, et al: The neurovascular outcome of scapulothoracic dissociation. J Vasc Surg
1993;17:1083-1088.
 Oreck SL, Burgess A, Levine AM: Traumatic lateral displacement of the scapula: A radiographic sign of neurovascular
disruption. J Bone Joint Surg Am 1984;66:758-763.

90. A 21-year-old man who underwent repair of a distal biceps tendon rupture using a
two-incision approach 4 months ago now reports difficulty gaining rotation of his
forearm. Figures 49a and 49b show the AP and lateral radiographs. What is the
most likely cause of his problem?

1- Inadequate physical therapy


2- Exposure of the periosteum of the lateral
ulna during surgery
3- Inappropriate location of the suture
anchor
4- Fixation of the tendon with the forearm
fully pronated
5- Subluxation of the radial head
PREFERRED RESPONSE: 2

DISCUSSION: The radiographs show early ectopic bone formation originating between the ulna
and the radius. The development of ectopic bone in this area following a two-incision approach
for anatomic repair of the distal biceps tendon is thought to be related to exposure of the
periosteum of the lateral ulna during surgery. This can be avoided by the use of a muscle-
splitting incision between the extensor carpi ulnaris and common extensor muscles. Full
pronation of the forearm allows for the necessary exposure of the radial tuberosity during the
procedure and for fixation of the tendon at its maximal length.

 Morrey BF: Tendon injuries about the elbow, in Morrey BF (ed): The Elbow and Its Disorders, ed. 2. Philadelphia, PA, WB
Saunders, 1993, pp 492-503.
 Failla JM, Amadio PC, Morrey BF, Beckenbaugh RD: Proximal radioulnar synostosis after repair of distal biceps brachii
rupture by the two-incision technique: Report of four cases. Clin Orthop 1990;253:133-136.

91. A 53-year-old man reports acute, severe left shoulder pain after undergoing
abdominal surgery 10 days ago. Initial management, consisting of anti-
inflammatory drugs, physical therapy, and a subacromial injection of corticosteroid,
fails to provide relief. Reexamination of the shoulder 2 months after the onset of
symptoms reveals atrophy of the infraspinous and supraspinous fossa and profound
weakness of active abduction and external rotation. His neck is supple with a full
range of motion. Plain radiographs and an MRI scan of the shoulder are normal.
What diagnostic study should be performed next in the evaluation of this patient?

1- Shoulder arthrography
2- MRI of the cervical spine
3- CT of the head
4- Technetium Tc 99m bone scan
5- Electromyography and nerve conduction velocity studies

PREFERRED RESPONSE: 5

DISCUSSION: Suprascapular nerve palsy is a fairly uncommon yet well-known cause of


shoulder pain and weakness. A variety of causes have been described, including compression by
a ganglion cyst, an anomalous or thickened superior transverse scapular ligament, a humeral and
scapular fracture, and traction or kinking of the nerve in the suprascapular notch.
In this patient, the injury is most likely caused by traction or compression of the nerve in the
suprascapular notch as the result of positioning during abdominal surgery; therefore, the studies
of choice are electromyography and nerve conduction velocity studies. While MRI of the
cervical spine may be of some value in ruling out a radiculopathy, the clinical history does not
support such a cause for this condition.

 Rengachary SS, Neff JP, Singer PA, Brackett CE: Suprascapular entrapment neuropathy: A clinical, anatomical, and
comparative study. Part 1: Clinical study. Neurosurgery 1979;5:441-446.
 Rengachary SS, Burr D, Lucas S, Hassanein KM, Mohn MP, Matzke H: Suprascapular entrapment neuropathy: A clinical,
anatomical and comparative study. Part 2: Anatomical study. Neurosurgery 1979;5:447-451.

Dr. Ahmed Altaei


American Academy of Orthopaedic Surgeons
2002 shoulder Self-Assessment Examination

 Bigliani LU, Dalsey RM, McCann PD, April EW: An anatomical study of the suprascapular nerve. Arthoscopy 1990;6:301-
305.

92. A 58-year-old reports pain and stiffness in his left shoulder following a seizure
Page | 65
episode. Diagnosis at the time of the seizure is a frozen shoulder, and management
consists of an aggressive physical therapy program of stretching exercises. Four
months later he continues to have shoulder pain and has not gained any additional
range of motion. A CT scan is shown in Figure 50. Management should now consist
of

1- additional physical therapy


and home stretching
exercises.
2- closed reduction and
immobilization in a spica
cast.
3- open reduction and transfer of
the subscapularis and lesser
tuberosity.
4- humeral arthroplasty.
5- total shoulder arthroplasty.

PREFERRED RESPONSE: 4

DISCUSSION: Humeral arthroplasty is indicated for chronic posterior dislocations when the
impression defect in the humeral head is greater than 45% to 50%. If the condition remains
undiagnosed for more than 9 to 12 months, secondary degenerative changes on the glenoid may
occur, necessitating total shoulder arthroplasty. Open reduction and transfer of the subscapularis
and lesser tuberosity are used for impression defects that consist of 20% to 40% of the humeral
articular surface. Closed reduction and immobilization with the arm in slight extension and
external rotation is useful when the posterior dislocation is diagnosed within the first 6 weeks
and the articular defect is less than 20%.

 Hawkins RJ, Neer CS II, Pianta RM, Mendoza FX: Locked posterior dislocation of the shoulder. J Bone Joint Surg Am
1987;69:9-18.
 Checchia SL, Santos PD, Miyazaki AN: Surgical treatment of acute and chronic posterior fracture-dislocation of the
shoulder. J Shoulder Elbow Surg 1998;7:53-65.

93. When conducted at near physiologic strain rates, tensile studies of the inferior
glenohumeral ligament (IGHL) have shown that the

1- anterior band of the IGHL has the greatest stiffness and the glenoid insertion site
shows greater strain than the ligament midsubstance.
2- anterior band of the IGHL has the greatest stiffness and the ligament
midsubstance shows greater strain than the glenoid insertion site.
3- axillary pouch of the IGHL has the greatest stiffness and the glenoid insertion site
shows greater strain than the ligament midsubstance.
4- axillary pouch of the IGHL has the greatest stiffness and the ligament
midsubstance shows greater strain than the glenoid insertion site.
5- posterior portion of the IGHL has the greatest stiffness and the glenoid insertion
site shows greater strain than the ligament midsubstance.

PREFERRED RESPONSE: 1

DISCUSSION: Tensile testing of the inferior glenohumeral ligament at near physiologic strain
rates has shown that the anterior band of the IGHL has the greatest stiffness of the three ligament
regions and the glenoid insertion site shows greater strain than the ligament midsubstance.

 Bigliani LU, Pollock RG, Soslowsky LJ, Flatow EL, Pawluk RJ, Mow VC: Tensile properties of the inferior glenohumeral
ligament. J Orthop Res 1992;10:187-197.
 Ticker JB, Bigliani LU, Soslowsky LJ, Pawluk RJ, Flatow EL, Mow VC: Inferior glenohumeral ligament: Geometric and
strain-rate dependent properties. J Shoulder Elbow Surg 1996;5:269-279.

94. Manipulation under anesthesia for resistant frozen shoulder should be avoided in
patients with

1- idiopathic onset.
2- gout.
3- hyperthyroidism.
4- hypothyroidism.
5- severe osteoporosis.

PREFERRED RESPONSE: 5

DISCUSSION: Severe osteoporosis is a contraindication to manipulation under anesthesia in


patients with a resistant frozen shoulder because of the higher risk of humeral fracture.
Manipulation is considered for frozen shoulder in patients who are symptomatic despite
undergoing a reasonable course of appropriate physical therapy.

 Harryman DT II: Shoulder: Frozen and stiff. Instr Course Lect 1997;42:247-257.
 Warner JJ: Frozen shoulder: Diagnosis and management. J Am Acad Orthop Surg 1997;5:130-140.

95. A patient who sustained a cerebrovascular accident (CVA) 18 months ago has a
long-standing spastic adduction contracture of the shoulder with a rigid block to
passive external rotation. Significant hygiene problems exist with maceration and
continued skin breakdown. Management should consist of

1- a percutaneous pectoralis tenotomy.

Dr. Ahmed Altaei


American Academy of Orthopaedic Surgeons
2002 shoulder Self-Assessment Examination

2- a modified L’Episcopo procedure.


3- serial lidocaine nerve blocks.
4- pectoralis tenotomy and subscapularis tendon lengthening.
5- phenol nerve blocks.

PREFERRED RESPONSE: 4 Page | 67

DISCUSSION: Following a CVA, the muscular imbalance often leads to a fixed contracture of
the shoulder in adduction, internal rotation, and flexion. The responsible muscles include the
pectoralis major, subscapularis, teres major, and latissimus dorsi. If stretching cannot produce
enough improvement for axillary hygiene, then surgery is an option. If the shoulder resists
external rotation during examination with the arm at the side, as in this patient, then the
subscapularis is spastic and contributing to the deformity as well and needs to be released along
with the pectoralis. Phenol nerve blocks are most effective and best given within 6 months of the
initial CVA to be effective. Lidocaine blocks may be helpful in determining whether a deformity
is caused by a fixed soft-tissue contracture or by spasticity but play no role once the contracture
is present. The modified L’Episcopo procedure is indicated in patients with contracture
secondary to brachial plexus birth palsies.

 Braun RM, Botte MJ: Treatment of shoulder deformity in acquired spasticity. Clin Orthop 1999;368:54-65.
 McCollough NC III: Orthopaedic evaluation and treatment of the stroke patient. Instr Course Lect 1975;24:45-55.

96. A patient with degenerative osteoarthritis of the sternoclavicular (SC) joint reports
constant pain, discomfort, and marked prominence and instability of the SC joint
following medial clavicle resection. Which of the following procedures is most likely
to produce these signs and symptoms?

1- Excision medial to the costoclavicular ligament


2- Excision lateral to the costoclavicular ligament
3- Excision of the coracoclavicular ligaments and lateral clavicle
4- Excision of the coracohumeral ligaments
5- Leaving the costoclavicular ligament intact

PREFERRED RESPONSE: 2

DISCUSSION: Medial clavicle excision alone can be associated with postoperative instability of
the clavicle. The clavicle should be stabilized to the first rib by reconstructing the
costoclavicular ligament if it is torn or if the resection is lateral to its clavicular insertion.
Therefore, care must be taken to resect only that part of the clavicle that is medial to the
costoclavicular ligament. Adequate protection for vital structures that lie posterior to the medial
end of the clavicle must be provided.

 Bremner RA: Nonarticular noninfected subacute arthritis of the sternoclavicular joint. J Bone Joint Surg Br 1959;41:749-
753.
 Rockwood CA Jr: Disorders of the sternoclavicular joint, in Rockwood CA Jr, Matsen FA III (eds): The Shoulder.
Philadelphia, PA, WB Saunders, 1998, vol 1, pp 583-586.
97. A 26-year-old man has had a 2-year history of pain and stiffness after sustaining a
comminuted olecranon fracture. Treatment at the time of injury consisted of open
reduction and internal fixation with tension band wiring. Examination reveals
motion of 45 degrees to 110 degrees and pain throughout the arc of motion.
Resisted flexion and extension are painful. Forearm rotation is normal.
Radiographs are shown in Figure 51. Treatment should consist of

1- excision of heterotopic bone.


2- hardware removal and elbow joint
release with splinting.
3- semiconstrained total elbow
arthroplasty.
4- distraction arthroplasty.
5- synovectomy and radial head
excision.

PREFERRED RESPONSE: 2

DISCUSSION: The patient has posttraumatic arthritis of the elbow; therefore, the treatment of
choice is hardware removal and soft-tissue releases with splinting to avoid recurrence of
contractures. The combination of pain and stiffness in an elbow that has sustained significant
joint surface damage renders it unresponsive to simple soft-tissue releases and heterotopic bone
excision. Joint distraction and interposition arthroplasty offer the possibility of maintaining
motion and relieving pain as a later salvage procedure. Joint replacement should not be
performed in young, active, strong individuals because the prosthesis will fail quickly and
complications will develop. Synovectomy and radial head excision are not indicated.

 Morrey BF: Distraction arthroplasty: Clinical applications. Clin Orthop 1993;293:46-54.


 O’Driscoll SW: Elbow arthritis: Treatment options. J Am Acad Orthop Surg 1993;1:106-116.

98. What is the most common cause of rotator cuff injury in high school athletes?

1- A curved or type III acromion


2- A tight coracoacromial ligament
3- Overuse
4- Limited internal rotation
5- Scapulothoracic dyskinesia

PREFERRED RESPONSE: 3

Dr. Ahmed Altaei


American Academy of Orthopaedic Surgeons
2002 shoulder Self-Assessment Examination

DISCUSSION: A large number of etiologies of rotator cuff injury have been proposed. Both
intrinsic and extrinsic mechanisms have been suggested. In the young athlete the common
underlying mechanism is overuse. Contributing factors include increased laxity, anatomic
variation in the coracoacromial arch, and altered kinematics.

 Wilkins KE: Shoulder injuries: Epidemiology, in Stanitski CL, DeLee JC, Drez D Jr (eds): Pediatric and Adolescent Sports Page | 69
Medicine. Philadelphia, PA, WB Saunders, 1994, pp 175-182.
 Sher JS: Anatomy, biomechanics, and pathophysiology of rotator cuff disease, in Iannnotti JP, Williams GR (eds): Disorders
of the Shoulder: Diagnosis and Management. Philadelphia, PA, Lippincott Williams and Wilkins, 1999, pp 3-30.

99. A 16-year-old boy with osteochondritis dissecans of the capitellum has intermittent
symptoms of catching and locking. Examination is unremarkable. Radiographs
reveal a loose body anteriorly with a diameter of 10 mm. To remove the loose body,
elbow arthroscopy is being considered. Which of the following procedures would
minimize the risk of neurovascular complication during the procedure?

1- Keeping a smooth plastic cannula in each portal after it is established


2- Using an image intensifier to localize the loose body
3- Distending the elbow joint capsule prior to establishing the anterolateral portal
4- Placing the scope in the proximal anteromedial portal and then enlarging the
anterolateral portal so that it is bigger than the maximum diameter of the loose
body
5- Breaking up the loose body into several pieces prior to extracting it

PREFERRED RESPONSE: 3

DISCUSSION: Complications of elbow arthroscopy are usually minor or temporary. However,


serious complications include nerve injuries. The deep radial nerve is the closest to any of the
portals, resting as close as 1 mm away from the scope inserted in the anterolateral portal. The
capsule can be displaced anteriorly by distending the joint with about 25 mL of saline solution,
thus moving the deep radial nerve approximately 1 cm anteriorly and decreasing the risk of
injuring it while establishing the anterolateral portal. Keeping plastic cannulae in the portals
may help to diminish fluid extravasation and swelling, which is more of an impediment than a
serious complication. The image intensifier has no documented role in guiding loose body
removal. While the proximal anteromedial portal is probably the safest anterior portal to
establish, it is actually easier to remove a large loose body from this portal while viewing it from
an anterolateral position. There is less tendon and muscle bulk to pass through at the site of the
proximal anteromedial portal than at the anterolateral portal, making it less likely for the loose
body to get stuck in the soft tissues. Techniques have been developed to permit removal of loose
bodies as large as 2 cm in diameter without breaking them up into pieces. If it is possible to
remove a large loose body intact, doing so greatly simplifies and shortens the procedure.

 Lynch GJ, Meyers JF, Whipple TL, Caspari RB: Neurovascular anatomy and elbow arthroscopy: Inherent risks.
Arthroscopy 1986;2:190-197.
 O’Driscoll S: Loose bodies and synovial conditions, in Green D, Hotchkiss R, Pederson W (eds): Green’s Operative Hand
Surgery. New York, NY, Churchill Livingstone, 1999, pp 235-249.
100. Examination of the shoulder seen in Figure 52 shows atrophy and tenderness of the
infraspinous fossa and profound weakness in external rotation. The supraspinous
fossa shows normal muscle bulk. What is the most likely cause of this condition?

1- Neurofibroma of the suprascapular


nerve
2- Ganglion cyst of the suprascapular
notch
3- Ganglion cyst of the spinoglenoid
notch
4- Lipoma of the suprascapular notch
5- Lipoma of the spinoglenoid notch

PREFERRED RESPONSE: 3

DISCUSSION: Compression of the suprascapular nerve by a ganglion cyst is a well-documented


cause of pain and weakness in the shoulder. Isolated involvement of the infraspinatus indicates
that the area of entrapment is at the spinoglenoid notch and not the suprascapular notch. The
majority of ganglion cysts found in the shoulder are related to tears of the labrum. When such a
compressive lesion is found, decompression can be accomplished through either an open or
arthroscopic approach. Several authors have shown the value of arthroscopy in the treatment of
this condition. It has been shown that it is technically possible to decompress a paralabral
ganglion cyst using arthroscopy; this method is usually followed by repair of the torn labrum.
Alternatively, arthroscopic repair of the labrum can be performed and the cyst may be aspirated
at the time of surgery. Open cyst excision through a posterior approach is also an acceptable
method of treatment.

 Schickendantz MS, Ho CP: Suprascapular nerve compression by a ganglion cyst: Diagnosis by magnetic resonance imaging.
J Shoulder Elbow Surg 1993;2:110-114.
 Thompson RC, Schneider W, Kennedy T: Entrapment neuropathy of the inferior branch of the suprascapular nerve by
ganglia. Clin Orthop 1982;166:185-187.
 Iannotti JP, Ramsey ML: Arthroscopic decompression of a ganglion cyst causing suprascapular nerve compression.
Arthroscopy 1996;12:739-745.

Dr. Ahmed Altaei

You might also like